Jump to content

Wikipedia:Reference desk/Miscellaneous

From Wikipedia, the free encyclopedia

This is an old revision of this page, as edited by MalwareSmarts (talk | contribs) at 21:16, 14 April 2008 (Building an instrument). The present address (URL) is a permanent link to this revision, which may differ significantly from the current revision.

Welcome to the miscellaneous section
of the Wikipedia reference desk.
Select a section:
Want a faster answer?

Main page: Help searching Wikipedia

   

How can I get my question answered?

  • Select the section of the desk that best fits the general topic of your question (see the navigation column to the right).
  • Post your question to only one section, providing a short header that gives the topic of your question.
  • Type '~~~~' (that is, four tilde characters) at the end – this signs and dates your contribution so we know who wrote what and when.
  • Don't post personal contact information – it will be removed. Any answers will be provided here.
  • Please be as specific as possible, and include all relevant context – the usefulness of answers may depend on the context.
  • Note:
    • We don't answer (and may remove) questions that require medical diagnosis or legal advice.
    • We don't answer requests for opinions, predictions or debate.
    • We don't do your homework for you, though we'll help you past the stuck point.
    • We don't conduct original research or provide a free source of ideas, but we'll help you find information you need.



How do I answer a question?

Main page: Wikipedia:Reference desk/Guidelines

  • The best answers address the question directly, and back up facts with wikilinks and links to sources. Do not edit others' comments and do not give any medical or legal advice.
See also:


April 8

How do you find and contact an editor?

I love writing, and I want to go professional some day, but I don't know HOW! Can you help me? —Preceding unsigned comment added by The'yellow'poet (talkcontribs) 00:38, 8 April 2008 (UTC)[reply]

What type of writing? It is different for different genres. But generally you're best off if you find a literary agent first. They know how to contact editors, it's their job to make it work out. --Captain Ref Desk (talk) 00:54, 8 April 2008 (UTC)[reply]
I recommend that you buy, borrow or steal a dictionary, read a lot of good writers, and look up the words that you don't know, such as the word dubious (which you at least know how to spell), before you seriously consider a career in writing.  --Lambiam 05:48, 8 April 2008 (UTC)[reply]

Or do what I did. I started two articles and got them published in an unknown non famous magazine, but it still got my name out. Then I was basically allowed to write whatever of course in time lines. I'm suppose to finish my novel in six months. Its suppose to be out this year. The problem is I never can stick to one idea. I have so many ideas in my head and can never tie myself down properly. Dubious unsure about the outcome. Dubious possible dishonest or immoral. Dubious of uncertain quality. This is what I remember on top of my head. I have a whole bunch of notes on spelling alternate spellings. Simple definitions of even the most complex word. The only thing I can't seem to get down is math.

Always

Cardinal Raven

Cardinal Raven (talk) 06:17, 8 April 2008 (UTC)Cardinal Raven[reply]

Worry not, that's why there's the mathematics refdesk (though they may not be interested in turning it into plain english for you)  ; ) Julia Rossi (talk) 08:59, 8 April 2008 (UTC)[reply]

Hi yellow, can you be more specific: do you want to publish short / long fiction ? or periodical writing such as journalism, comment etc in magazines, newspapers and so on ? The requirements can be quite different. 125.237.88.118 (talk) 11:47, 8 April 2008 (UTC)[reply]

Suleiman the Magnificent

habidashery of Suleiman the Magnificent

What on earth do we call that hat? Besides "enormous"? --Masamage 05:34, 8 April 2008 (UTC)[reply]

At[1] they say it's a 16th century Ottoman turban. And if your name is "the Magnificient" it's not going to be street-wear, it's going to be toweringly regal (regalia?). As far as I can discover, it varied from showing much of the inner cone with much less swathing to this. (The earlier headgear with the fur trim c. 13th c's called a sharbush and looks more tribal.) Julia Rossi (talk) 05:50, 8 April 2008 (UTC)[reply]
(after edit conflict) It is a turban. See Turban#History of turbans. This shape may help to explain the etymology of the word tulip, from older Turkish tülbent "turban". (In modern Turkish, the word is türban.)  --Lambiam 05:58, 8 April 2008 (UTC)[reply]
I knew it was some kind of turban, but they're mostly small and modest. Still, useful information there. Is there a specific name for a huge gigantic turban? --Masamage 06:03, 8 April 2008 (UTC)[reply]
A very informal, possibly insulting name is "onion head hat." Adam Bishop (talk) 07:42, 8 April 2008 (UTC)[reply]
As a resident of the city which stopped good old Suleiman´s progress in 1529 (at which time I, the magnificent Cookatoo single-handedly defended the occident from the invading infidels), let me direct you to http://www.sikhnet.com/s/tyingturbans. There are a few different names for the various ways to tie a Sikh turban.
Oddly enough, whilst many of the Turkish / Muslim women in the Vienna of today wear head scarves or even veils, I have never seen a male wearing a turban. Well, I have, but they were clearly Indian Sikhs. In our article on turban there is a picture of a Sikh wearing a rather large turban, but no name is given to it. --Cookatoo.ergo.ZooM (talk) 08:47, 8 April 2008 (UTC)[reply]
"Small and modest"? Massamage I take it you haven't been hanging with a 16th century sovereign suzerain with the power of life and death lately. Looking at the visuals, Suleiman's hat really stood out (not to mention his helmet), which is the point, isn't it?  ; ) Julia Rossi (talk) 08:53, 8 April 2008 (UTC)[reply]
I said most turbans are small and modest--eg. the ones I see Sikhs wearing around. --Masamage 21:27, 8 April 2008 (UTC)[reply]
tughra of Suleiman the Magnificent

PS Can't wait for someone to suggest "Mother of all turbans"?

Perhaps 'the Turbanator'? ;) Lemon martini (talk) 12:11, 8 April 2008 (UTC)[reply]

Hey, you think his turban is ostentatious, check out his tughra:
Steve Summit (talk) 18:30, 8 April 2008 (UTC)[reply]
I believe the theory of "Turban Envy", as proposed by a late countryman of mine has some bearing on this disturban topic.
Also bear in mind that ornate feathery headgear amongst Amerindians, top hats or the pope´s tiara were / are reserved for (the) hoi polloi. Our article on papal tiara, by the way, mentions that Suleiman had his own tiara made. In a show of one-upmanship it had one more tier than the "small and modest" ones in Rome. Clearly, StM knew about the power of status symbols and the importance of carrying a big stick. --Cookatoo.ergo.ZooM (talk) 18:59, 8 April 2008 (UTC)[reply]
You must mean the "hoity toity" since hoi polloi is the rabble, the masses, the rif raf.  ; ) Julia Rossi (talk) 22:55, 8 April 2008 (UTC)[reply]
Oooooooooooops, it´s all Greek to me. --Cookatoo.ergo.ZooM (talk) 23:34, 8 April 2008 (UTC)[reply]
Say, I wander around Turkey. Can I buy one somewhere? 24.130.192.59 (talk) 22:13, 8 April 2008 (UTC)[reply]

"Say, I wander around Turkey. Can I buy one somewhere?"

You're going to have to buy me as well (I don't know how I'm going to get from you). Cause the only people who wear those kind of hates are all sorts of modest.71.142.208.226 (talk) 22:21, 8 April 2008 (UTC)Cardinal Raven[reply]

It won't look like that until you put the kit together. I'm guessing it's one extra long piece of fabric with an inner cone and you'll need lessons. This[2] looks quite the rival. Julia Rossi (talk) 09:34, 9 April 2008 (UTC)[reply]
Here's a sizable female version. Similar to Julia Rossi's towering example is this turban. ---Sluzzelin talk 11:03, 9 April 2008 (UTC)[reply]
Well done, Sluzzelin, a top hat. Presenting Major Singh, weighing in at 68 lbs, steel cables and 400 meters of cloth – now this is a hat!! : )) Julia Rossi (talk) 12:07, 9 April 2008 (UTC)[reply]
... and it ain't no slouch either. :-| ---Sluzzelin talk 13:58, 9 April 2008 (UTC)[reply]
By the way, the turban seems to have gone out of Ottoman royal fashion sometime in the early 19th century. All the Sultans up to Mustafa IV are depicted wearing a turban. The succeeding Sultans, starting with Mahmud II, are shown wearing a fez, (though Mahmud II can be seen wearing both).
From the article on the Fez: "During the reign of the Sultan Mahmud Khan II (1808-39), a European code of dress gradually replaced the traditional robes worn by members of the Ottoman court. The change in costume was soon emulated by the public and senior civil servants, followed by the members of the ruling intelligentsia and the emancipated classes throughout the Ottoman Empire. While European style coats and trousers were gradually adopted, this change did not extend to headwear. Peaked or broad brimmed headdresses such as the top hat did not meet the Islamic requirement that men should press their heads to the ground when praying. Accordingly the Sultan issued a firman (royal decree) that the checheya headgear in a modified form would become part of the formal attire of the Turkish Empire irrespective of his subjects' religious sects or millets."---Sluzzelin talk 13:58, 9 April 2008 (UTC)[reply]

Habits

How much gum would you have to chew to strain your jaw out? What are the long terms effects of biting the inner lip? This is all out of my own curiosity.


Thank You

Always

Cardinal Raven

Cardinal Raven (talk) 06:13, 8 April 2008 (UTC)Cardinal Raven[reply]

The inner lip thing sounds like self-harm so mentally, long-term, not great. Julia Rossi (talk) 09:35, 8 April 2008 (UTC)[reply]
I assume he means accidentally biting the inside of the lip or cheek. StuRat (talk) 11:04, 8 April 2008 (UTC)[reply]
(Oops, blinded by visions of bleeding lips, tks StuRat) Julia Rossi (talk) 22:45, 9 April 2008 (UTC)[reply]
you'd have to chew a lot, my friend. the human jaw is designed for chewing so its gonna be hard to over strain that. but possible, as I said. you'd just have to chew a lot, thats all. —Preceding unsigned comment added by 87.230.6.32 (talk) 14:04, 8 April 2008 (UTC)[reply]

To strain out your jaw I think you would have to stuff more gum in your mouth than your mouth is used to chewing at one time. Your jaw will start to hurt in no time. I don't recommend doing it though.--Dlo2012 (talk) 14:38, 8 April 2008 (UTC)[reply]

"Celery, raw, develops the jaw/But celery, stewed, is more quietly chewed." --Ogden Nash. No doubt the same principle applies to gum. Human muscles are meant to be used. BrainyBabe (talk) 17:55, 8 April 2008 (UTC)[reply]

Well its not entirely accidental biting the lip. When I was younger I had fallen and had bitten my inner lip and cut the top of my upper lip (did not feel to good). After that the inner lip in front of the two front teeth has a lump that is virtually numb of any pain. I chew that all the time. That is why I chew gum to get my mind off of chewing the lump. I chew a lot of gum I go through one pack of gum in one day. The same thing kinda goes for peas. I eat five cans of peas a day. I hate celery. It taste bad cook or stewed. Blugh, celery. So more or less I was curious to how much gum would strain a jaw? And the effects of biting a lump more or less.

Always

Cardinal Raven

71.142.208.226 (talk) 22:16, 8 April 2008 (UTC)Cardinal Raven[reply]

Intentionally chewing on that lump could cause you all sorts of problems, from infection to potentially cancer. Don't do it. StuRat (talk) 16:01, 10 April 2008 (UTC)[reply]

background to name of a plant (lily)

I have come across a lily called 'elaine garrad'. I would intersted to know how this name came aboutPathumfrey (talk) 12:57, 8 April 2008 (UTC)[reply]

I searched around a bit, and couldn't find any specific details on that cultivar. In general, though, it seems that the person who has created and registered a particular cultivar or hybrid gets to select the name. As an example, look at this page showing hybrids; the name of the hybrid is shown, along with the person who registered it. --LarryMac | Talk 13:36, 8 April 2008 (UTC)[reply]
I don't know anything about lilies, but perhaps this link will get you started. It's the entry for the 'Elaine Garrad' cultivar in the Royal Horticultural Society's Lily Register. I'm not entirely sure of the notation, but it appears that Elaine Garrad was registered in 1984 by D.S. Marriott. There's no indication how the name was selected, but perhaps that will get you started on your search. TenOfAllTrades(talk) 13:50, 8 April 2008 (UTC)[reply]

fake?

this is fake, right? [3] —Preceding unsigned comment added by 128.117.43.34 (talk) 13:57, 8 April 2008 (UTC)[reply]

Nope, it is absolutely genuine, and can be found in London, on the Isle of Dogs, close to Canary Wharf. —Preceding unsigned comment added by Samilong (talkcontribs) 14:03, 8 April 2008 (UTC)[reply]

The picture is genuine, though it is not a real traffic signal but a sculpture[4]. -- Q Chris (talk) 14:44, 8 April 2008 (UTC)[reply]

Haley Williams haircut

How would you describe Haley Williams from paramores haircut? What should I ask for at the barbers if I want one the same? ps I'm a guy. —Preceding unsigned comment added by 212.159.16.175 (talk) 17:57, 8 April 2008 (UTC)[reply]

Why not just take in a photograph of the haircut and say "I'd like one of these, please" ? Samilong (talk) 18:07, 8 April 2008 (UTC)[reply]
Have at look at this yahoo! answers post. Has some good pictures of her hair. D0762 (talk) 18:12, 8 April 2008 (UTC)[reply]
Ok, but as a guy don't you think they might act a bit funny if I take a pic of a girl and say one of these, please? Won't they be like "oh, so he's sailing down THAT side of the river, is he"? —Preceding unsigned comment added by 212.159.16.175 (talk) 18:56, 8 April 2008 (UTC)[reply]
People have all sorts of haircuts and in modern times the stereotypical gender barriers have mostly fallen away. Of course you may experience some discrimination, especially in small-town middle America, but the most important thing is to feel secure and confident in yourself. Have a read of this. D0762 (talk) 19:22, 8 April 2008 (UTC)[reply]

Yeah go for that haircut. Really I don't think people will look at you weirdly. Haircuts like that are wildly excepted for some reason. One time just for fun (Halloween costume party fun) I shave parts of my hair to make go bald and then I did a comber over and dress up like Trump. Oh, that was good feeling.71.142.208.226 (talk) 22:19, 8 April 2008 (UTC)Cardinal Raven[reply]

Appraisal

What's the value of a good condition vintage '79 Peavey 400 Mark III half stack? Bellum et Pax (talk) 19:07, 8 April 2008 (UTC)[reply]


i would say about 100 pounds or so, it depends where you are, and what sort of condition its in, it must work, and look nice, people are shallow —Preceding unsigned comment added by 86.18.34.51 (talk) 20:28, 8 April 2008 (UTC)[reply]
For the sake of those who have no idea what you're talking about, what is a vintage '79 Peavey 400 Mark III half stack? Astronaut (talk) 23:39, 8 April 2008 (UTC)[reply]

From the sounds of it, I'm guessing some sort of guitar amp. Acceptable (talk) 23:50, 8 April 2008 (UTC)[reply]

Something you Cant have

what is the word that is desribing a state to were you want something so bad and you know you cant have it —Preceding unsigned comment added by 24.149.90.98 (talk) 20:25, 8 April 2008 (UTC)[reply]

may be related to the law of the forbidden, one where one wants something, because you cant have it, eg I can get almost any woman I want, but the only ones I want are the ones that dont want me —Preceding unsigned comment added by 86.18.34.51 (talk) 20:30, 8 April 2008 (UTC)[reply]

I'm really not sure but "covet" might cover it, sort of. Like you'd "covet your neighbour's car" because you know you can;t have it. You'd get better luck asking on Languages 81.96.161.104 (talk) 22:31, 8 April 2008 (UTC)[reply]

Desire. Sounds awfully simple, I know, but if you already have something there's no desire. Vranak (talk) 22:41, 8 April 2008 (UTC)[reply]

Crave or craving, is like covet. Julia Rossi (talk) 22:48, 8 April 2008 (UTC)[reply]
Envy?HS7 (talk) 11:24, 13 April 2008 (UTC)[reply]

zoom

i am trying to find a Zoom UIB02 USB Interface Board for a zoom mrs 1608 multitrack recorder, does any one know where i can get one, how much they cost, and what program is used on ones computer. Thanks —Preceding unsigned comment added by 86.18.34.51 (talk) 20:27, 8 April 2008 (UTC)[reply]

It would probably be best to ask the computer desk (here). They deal with most technological stuff in general, despite the name.--LaPianísta! 17:23, 9 April 2008 (UTC)[reply]

Editing

Will wikipedia be edited by professionals in the future? —Preceding unsigned comment added by 35.11.38.122 (talk) 23:12, 8 April 2008 (UTC)[reply]

Professional bowlers. 200.127.59.151 (talk) 23:36, 8 April 2008 (UTC)[reply]
I wish I was a professional bowler. I average about 175. Useight (talk) 23:40, 8 April 2008 (UTC)[reply]
I, too, am a newb at bowling. Neal (talk) 23:49, 8 April 2008 (UTC).[reply]

I think in a way Wikipedia is all ready edited by professionals. Sometimes people know subjects because they are so passionate about them. Video games are edited and written about and researched about because the person playing them was passionate about it. Then they created an article on Wikipedia because of all their passionate information. Wikipedia is all ready a community of professionals yes some of them might just be bums in their homes always on the computer or an actual professor that doesn't make them unprofessional. They are professional on what they know. That is what makes Wikipedia more human. And even professor need to brush up on their information. No one is ever right and in this day and age study, research and science always with new information about that subject. Information is always changing and our knowledge is alway getting bigger as well. The more we learn as unprofessionals the more we become professional.

Always

Cardinal Raven71.142.208.226 (talk) 00:41, 9 April 2008 (UTC)Cardinal Raven[reply]

I see what you're saying, that in a way it's edited by experts, at least in some fields, but it certainly not edited by profesionals, as all editing here is voluntary and therefore amateur, even if done by a professional researcher in that field as no original research, right? So although Wikipedia has many dedicated experts, it could never truly be wikipedia and be edited by professionals. 81.96.161.104 (talk) 01:05, 9 April 2008 (UTC)[reply]

I will beg to differ. Its edited by professionals. Yes, they may not be professionals in the way you look at it, but I like Wikipedia the way it is. Its more human the way it is now. I really couldn't sit here reading articles written by stiff logs (especially my video games). It would remind me of those times in the classroom trying to understand my monotonous teacher teach me something. And even true professionals aren't that professional. Later on in life the information they know may be expanded or even deleted for being false. There is never truly a professional the world is vast and ever changing some information may be erased and other information is put on.

Always

Cardinal Raven

71.142.208.226 (talk) 01:45, 9 April 2008 (UTC)Cardinal Raven[reply]

If the OP means will people be paid to edit the pedia, not while there's passion, knowledge and sharing is my guess. So, no. Julia Rossi (talk) 02:50, 9 April 2008 (UTC)[reply]
Cardinal Raven, that opinion is extremely naive. I hope the majority of readers don't want to read articles written by 14 year olds who think they are experts because they play a lot of video games. Adam Bishop (talk) 04:58, 9 April 2008 (UTC)[reply]

That is not what I meant. I don't want to know my articles are written by 14 years olds who think they are experts because they play a lot of video games. I don't want that to happen. No I meant that someone like me..when I like something say a video game I find as much information about that video games as I can. I find as much information. I want to learn about it. And I think that learning process and that knowledge is what makes people level up in information and knowledge holding. That is what I meant. Hope it made more sense now.71.142.208.226 (talk) 06:27, 9 April 2008 (UTC)Cardinal Raven[reply]

Imagine how much more successful in life you could be if you spent as much effort learning about useful things! (Wow, I'm really starting to sound like an old man.) Adam Bishop (talk) 07:52, 9 April 2008 (UTC)[reply]
It would surely be prohibitively expensive to pay professionals to write Wikipedia. And any professional worthy of the name wouldn't touch Wikipedia with a bargepole anyway. -88.110.156.249 (talk) 11:37, 9 April 2008 (UTC)[reply]
That's the first time I've heard anyone refer to real-life learning as a "level up". I may have to start using that one. 81.96.161.104 (talk) 13:40, 9 April 2008 (UTC)[reply]
I wonder if 35.11 might have meant "copy edited". Comparisons between Wikipedia and commercial reference works always touch upon how badly uneven the writing style and quality is, even where the factual accuracy is quite good.
However, it's hard to imagine the Foundation hiring enough copy editors to make a dent. *.Wikipedia.org is supposedly up to ten million articles. (Only about 25% in English!) Not to mention Wikibooks. APL (talk) 18:14, 9 April 2008 (UTC)[reply]

I've certainly edited article in areas where I worked as a professional , and I'm sure that is true of a great many editors, besides the Essjay type pseudo-professionals. Edison (talk) 04:37, 10 April 2008 (UTC)[reply]


April 9

The Good ol' Days of PEACE and QUIET!

My neighbors recently got a dog from the SPCA and for some reason whenever they leave the house their dog begins to bark immediately and constantly. I have complained to them and tried to ask for a solution. It has become such a problem where I have even taken my time to record the animal barking and had them listen to it. They still have done NOTHING about it. I have even talked to my apartment complex manager about and he has yet to sit down with them and talk to them about. I'm starting to wonder if I should just contact the Animal Control officers about it. What do you think would be the best course of action from here?--SlaveofBetrayal (talk)

Maybe they are trying to do something about it. Have you ever realized it takes a pretty damn hard time to train animal? Especially when the dog came from the SPCA and may have an attachment problem. I remember when I had a dog who came from the SPCA he got really lonely and he would bark like crazy. Its very difficult to train him that when I'm gone I'm gone. (That's why my neighbor ran him over.)

Always

Cardinal Raven

71.142.208.226 (talk) 02:35, 9 April 2008 (UTC)Cardinal Raven[reply]

Maybe it would have been more appropriate to tell how long this has been going on. Its been a 2 months since I last complained and I'm getting FED UP with it. What I asked for was a suggestion on how to handle it not a statement telling me about what YOU went through. I want ADVICE not childish banter.-- by SlaveofBetrayal (Talk) —Preceding comment was added at 03:08, 9 April 2008 (UTC)[reply]

Well maybe you should have been more specific you only gave us information as if its only been happening for a couple of weeks or one week, I'm sorry if it you saw my comment as childish banter, but it isn't. It was my advice for someone who had been experience this for a couple weeks. But in this case 2 months you should have called the animal control a long time. You should call the animal control.

Always

Cardinal Raven

71.142.208.226 (talk) 03:17, 9 April 2008 (UTC)Cardinal Raven[reply]

You might investigate what the laws are in your area. I have heard (but have never bothered to confirm) that in my local jurisdiction, the police can take action if a dog barks for more than 30 minutes continuously. Perhaps a visit from the police will help your neighbors understand why they need to take action, and soon?
Atlant (talk) 12:35, 9 April 2008 (UTC)[reply]

Why YouTube Doesn't Get Sued By Record Companies (cont.)

I hope it's okay that I added another comment down here (so people can see it and answer)along with the original comment that I'm responding to.

I am not a lawyer but, Apparently as a service provider they're (somewhat) immune to that sort of thing. Just as the phone company is not responsible for crimes committed over it's lines, and AOL isn't responsible for all crimes committed over AOL, YouTube is not necessarily responsible for the files on their servers, since they didn't put them there. (And they do make some small effort to get rid of copyright infringing files.) Whether or not this is true at all, or whether it applies to YouTube, and what countries it applies in are currently one giant, confusing mess. The Criticism of YouTube of article covers this a little, and points out that they have been sued for this sort of thing in the past. Also, remember that YouTube is now wholly owned by Google. APL (talk) 02:14, 7 April 2008 (UTC)

I remember the lawsuit by Viacom, but that was over television content. "We're losing money" was the argument they made with Napster, yet the same record companies look the other way when people upload any song they want to. Even stranger is that many record companies have accounts and upload thier own content, with no profit to be made there either. I bet that sometime soon YouTube will have every song ever released. That means you'll be able to hear any song you want any time you want (making me very happy!). What will happen then?...Sam Science (talk) 03:51, 9 April 2008 (UTC)[reply]
Another factor may be that Youtube works as free advertising for the recording artists in question. Although the files can be downloaded, these programs aren't a part of Youtube, and the sound quality of the files is normally awful. So although it works as a way for people to get the video, they are unlikely to replace record sales, and so pursuing the matter over Youtube may actually be harmful to the record companies. Why endanger such global free press and hype if the real impact on sales is being made through p2p and torrent networks? 81.96.161.104 (talk) 13:36, 9 April 2008 (UTC)[reply]

Ponzi scheme

Does Questnet Marketing chain come under Ponzi scheme?

```` —Preceding unsigned comment added by Aditya1986 (talkcontribs) 06:21, 9 April 2008 (UTC)[reply]

In my opinion, and in the opinion of several major governments (who have banned it), yes. See for example this set of reviews. (I could have sworn we had an article on it, but I can't find it.) —Steve Summit (talk) 13:01, 9 April 2008 (UTC) [P.S. it's four tildes, not four backtics. Use the Shift key. :-) ][reply]
Addendum: if it's not a Ponzi scheme, it's a pyramid scheme. —scs 13:09, 9 April 2008 (UTC)[reply]

can we sell my accounts?

I had a "blurtit" account & later on my sister also got a blurtit account. Now we have fully scored successfully running blurtit accounts..now we want to sell them in ebay or any other site. will anyone buy those accounts? 202.141.98.204 (talk) 06:56, 9 April 2008 (UTC)[reply]

When registration to blurtit is free, why on earth would anyone pay you guys for your accounts? That's just my opinion, but you might find some sucker out there so give it a go. Astronaut (talk) 13:55, 9 April 2008 (UTC)[reply]
No! U didn't got my sense fully...I'm in a hope that anyone would buy these accounts only b'coz registration on blurtit might be free for them but getting points in that won't be so easy. That too not just one or hundred or even thousand...now I had "3344" points to my credit & my sister has some 1500+ points...for getting these points it may take some 5-6 months for them...that's why i'm optimistic about the sale of both these accounts...now what do u say? 202.141.98.204 (talk) 06:00, 10 April 2008 (UTC)[reply]
Nice idea so, I googled Blurtit accounts for sale on... eBay, and Craig's list for examples but nothing. What I get is you get points for joining, sign up, log in, and being rated and other things. (And not sure, but do people sell on blurtit? ) Looking for a catch, I don't get if you get blocked for trying to sell them. Since you can ask on Blurtit, try asking another user if it's legit? Julia Rossi (talk) 06:12, 10 April 2008 (UTC)[reply]

Haley Williams haircut (cont.)

Ok, thanks for all the advice above. But what if I want to cut my own hair? How should I go about that without buchering myself or becming infamous as "that guy with the crap hair". Any good guides online I could download for cutting my own hair? Thanks all. —Preceding unsigned comment added by 212.159.16.175 (talk) 08:20, 9 April 2008 (UTC)[reply]

When I was a student with no money, I used to cut my own hair. After a wash, I would bend over the bath and cut it to roughly the same length all over. I never heard any comments, so I suppose if anyone thought I was "that guy with the crap hair", they were too polite to say. That said, if you want anything fancy, like Ms Williams' style, you are probably better off going to a professional salon (ie. not the local army barber :-). Astronaut (talk) 13:49, 9 April 2008 (UTC)[reply]

British telephone sockets... ARGGHHH!!

I'm trying to rewire the mess of wires in my basement after I moved to VoIP, and I am really confused at the colour coding there. Is there any difference between the British one and the New Zealand one? I have wires with 2 different schemes, one that has orange, blue and 2 identical white wires, while another with red, green, yellow, and black. Which ones are A&B lines? Is there some way I can test it to find out? --antilivedT | C | G 08:57, 9 April 2008 (UTC)[reply]

Can someone move this to technology please, it'll get a better answer! 86.139.92.22 (talk) 10:41, 9 April 2008 (UTC)[reply]

If you look carefully, you'll find that one of the white wires is twisted in a pair with the blue wire, the other with the orange wire. These form the first two pairs of the 25-pair color code (WHAAOE!). Pretend that one white wire has a blue stripe on it, the other an orange stripe (and indeed the paired wires are often striped that way, to make them easier to keep track of).
I believe the correspondence with the older green/red/black/yellow scheme is:
line 1 tip white/blue green
ring blue red
line 2 tip white/orange black
ring orange yellow
Steve Summit (talk) 12:50, 9 April 2008 (UTC) [edited 02:30, 10 April 2008 (UTC)][reply]
Of course, the colours of the insulation make absolutely no difference in practice - a straight wired cable is a straight wired cable... FiggyBee (talk) 21:19, 9 April 2008 (UTC)[reply]
Not sure what you mean by "in practice"! The colors can make quite a bit of difference:
  1. If the wiring will ever be worked on by other people, they will be very badly confused, and hate your guts (or worse) if you gratuitously use a non-standard color scheme. In electric power systems (if not telephony), using the standard colors for the hot, neutral, and ground wires isn't just a good idea, it's usually the law.
  2. In telephony and many other signal-transmission applications, the pairing truly matters. If you were to use (say) the blue and orange wires for one pair of a transmission circuit, and the two white wires for the other, it might not work well, or at all. (I still remember the first time I tried to wire up some RJ45 ethernet cables, and got the pairing wrong, and wondered why I was only getting 10mbps even though it was electrically a "straight wired cable".)
Steve Summit (talk) 02:30, 10 April 2008 (UTC)[reply]
That confirms my hypothesis: whoever that was wiring it in the first place didn't follow the standard. In the original mess I have one extension with blue/orange wired together and the 2 whites together, while in another one it's wired blue/white and red/white, and all of them only carry 1 phone line. Thanks anyway, now I need to make a loop circuit with a spare plug to see the wiring. --antilivedT | C | G 04:15, 10 April 2008 (UTC)[reply]

outdoor activities

what are dynamic activities? i've looked it up and cant seem to understand what they mean i can only get a rough idea like it means rock climbing ect. —Preceding unsigned comment added by 84.92.82.180 (talk) 10:10, 9 April 2008 (UTC)[reply]

What's the context it's being used in? Dismas|(talk) 14:08, 9 April 2008 (UTC)[reply]
I was about to ask the same thing. Generally and from a physiological angle, you could distinguish static activities, such as standing, sitting, and lying, from dynamic activities, such as walking, running, and swimming. If we're talking sports/leisure activities to begin with, maybe a static exercise machine could be compared to cycling, jogging, or rowing outdoors, where the environment changes and is dynamic. Then again, if it's from some sort of advertisement, dynamic could simply be a marketing word to make it sound more exciting and special. ---Sluzzelin talk 14:23, 9 April 2008 (UTC)[reply]

fake? (part 2)

Just a warning for those who may be at work or in the company of small children, the image is most likely NOT safe for work/children. I am not the OP but thought I should warn you. Dismas|(talk) 14:10, 9 April 2008 (UTC)[reply]

Please for the love of god tell me this image has been faked [5]— Preceding unsigned comment added by 212.159.16.175 (talk) 13:50, 9 April 2008 (UTC)[reply]

? Neal (talk) 13:58, 9 April 2008 (UTC).[reply]
Either a photoshop special, or a nasty accident in the sausage factory!! Astronaut (talk) 13:59, 9 April 2008 (UTC)[reply]

STUPID! Ignore the user who asked this question, he is a troll. Ericthebrainiac (talk) 14:25, 9 April 2008 (UTC)[reply]

Yes, definitely a fake, it's obvious that face and head have been pasted in. Richard Avery (talk) 14:41, 9 April 2008 (UTC)[reply]
Since the troll failed to amuse, here's a PBF comic that might. --Sean 19:47, 9 April 2008 (UTC)[reply]

education

can i do graduation after completing engineering.how can i take admission in IIT for graduation? by IIT-JEE or any other enterance test? —Preceding unsigned comment added by 117.198.80.137 (talk) 14:52, 9 April 2008 (UTC)[reply]

Did you google IIT-JEE[6]? it's all there. Julia Rossi (talk) 13:55, 10 April 2008 (UTC)[reply]

Body painting pictures

There has been talk at Talk:Body painting about a presentable top image that does not contain nudity. I remembered that at the 2007 World Bodypainting Festival, there was a female model who was painted in three different designs, all promoting Canon digital cameras, with both her genitals and breasts covered. But there is one thing keeping me from uploading the photographs I took of her, even if I get permission from the festival - Canon is a commercial, for-profit company, and by uploading these pictures I would be giving them free advertising. Is there any way I could get some sort of compensation from Canon? JIP | Talk 18:22, 9 April 2008 (UTC)[reply]

I doubt that Canon would be prepared to pay anyone, and I'm *certain* that Wikipedia wouldn't want to accept any money for advertising. That's a massive shit-storm that you don't want to start :-). If you can get the image under a free license then by all means use it, but don't try to have money change hands. 81.187.153.189 (talk) 18:36, 9 April 2008 (UTC)[reply]
I could get the image under a free licence just by asking the festival for permission, and if they give it, uploading the picture. But I feel that it would lose me the opportunity of earning money by advertising Canon. That's why I try not to take pictures where a commercial brand name is featured as the main subject. But this case was an exception because of the medium where the brand name was displayed. JIP | Talk 05:38, 10 April 2008 (UTC)[reply]
Have you thought of approaching Canon first? It seems like a process of elimination, Julia Rossi (talk) 22:52, 10 April 2008 (UTC)[reply]

Suzuki Wagon R+ (out of production)

Please don't scold - my wife has severe arthritis and when we bought this car we did so mainly because it was so easy for her to get into and out of - apart from it being so TALL with loads of headroom - and not forgetting it was so cheap - and has proved SO reliable during the past 4 years. But read any Car Review magazine and it rates lower than Noddy's car - no accounting for taste I suppose. But sadly, Suzuki have discontinued it and it has been re-badged as a Vauxhall Agila (UK). Any suggestions for a cheap town car with similar ease of entry, 5 doors, reliable, with air-conditioning, that is available in the UK, would be greatly appreciated. Thanks 92.8.31.214 (talk) 18:23, 9 April 2008 (UTC)[reply]

My mum is quite pleased with her Ford C-Max. Or you could take a look at Category:Compact MPVs. Astronaut (talk) 22:50, 9 April 2008 (UTC)[reply]

Weird question

My friend, who watches pornography way too much, told me that in German xxx movies there are almost no mammary intercourse scenes, meanwhile in American ones anal sex is rare (i.e. German, maybe whole European porn culture is built around anal-penetration-topics, but American porn industry interested in breast fetishism). Is it so? 89.146.66.238 (talk) 18:48, 9 April 2008 (UTC)[reply]

No idea. But German porn is WAY fucked up. On a side note, here is a pic (not work safe) you might find amusing = Image:Mamintb.PNG

That pic was not amusing. I am now traumatized for life. Thank you.71.142.208.226 (talk) 19:16, 9 April 2008 (UTC)Cardinal Raven[reply]

Seriously, that picture was creepy. Whoever drew that has waaaaay too much time on their hands. 81.96.161.104 (talk) 21:03, 9 April 2008 (UTC)[reply]

Uhm, back to the topic, please :) 89.146.66.238 (talk) 22:18, 9 April 2008 (UTC)[reply]

Back to the topic...anal sex is overwhelmingly common in recent American porn. I would suspect there were a hundred if not more anal sex scenes to every breast scene. Real breasts seem to have gone out of style in porn, replaced by fake boobs and very defined ribcages. --NellieBly (talk) 23:15, 9 April 2008 (UTC)[reply]

Europe

Could Europe be described as a "growth of peninsulas and islands sticking out of the west end of Asia"? —Preceding unsigned comment added by 212.159.16.175 (talk) 18:59, 9 April 2008 (UTC)[reply]

The language you have cited is a bit misleading. The word "growth", in particular, is really not right. Europe did not grow. It accreted. I would refer to it as a "region" a "continent" or "subcontinent" (depending on whether you consider Europe a continent unto itself or a part of Eurasia). It lies to the west of Asia, but the border between Europe and Asia is more of a side than a pointy end to my eyes. Marco polo (talk) 19:18, 9 April 2008 (UTC)[reply]
Fernand Braudel, in his two volume work "The Mediterranean", writes quite a bit of Europe as being a collection of peninsulas and isthmuses. He devotes quite a bit of text to the isthmuses, which he says are: the Russian isthmus (Baltic Sea to Black & Capsian Seas), the Polish isthmus ("Balkans to Danzig"), the German isthmus (North & Baltic Seas to Adriatic & Tyrrhenian Seas), and the French isthmus ("Rouen to Marseilles"). I've seen descriptions of Europe being a sort of peninsula elsewhere too. However, "growth" is an odd term to use. Pfly (talk) 17:33, 13 April 2008 (UTC)[reply]

Is it true?

Is it true that Serge Voronoff grafted monkey testicles onto human males. —Preceding unsigned comment added by 212.159.16.175 (talk) 19:06, 9 April 2008 (UTC)[reply]

If it's on Wikipedia, it must be true: Serge Voronoff. --Sean 19:59, 9 April 2008 (UTC)[reply]
Why ask when you can read the Serge Voronoff article? Is it because you're lazy? Or just trolling? By the way, the 1st sentence on the article, mentions monkey testicles. So there you have it. Neal (talk) 20:11, 9 April 2008 (UTC).[reply]
"Thin slices ... were implanted inside the patient's scrotum". Not quite the image that the question conjurs up. William Avery (talk) 21:17, 9 April 2008 (UTC)[reply]
Controversial answer for the hell of it: Yes, yes it's completely true, it's a fact based on years of reputable research and absolutely not involving slander, rumour or anything of the sort. But who hasn't grafted animal testes onto ahuman in this day and age? 81.96.161.104 (talk) 21:32, 9 April 2008 (UTC)[reply]
Wow it's actually true. I kind of assumed it wasn't. that's my learning for the day I guess. Crazy. 81.96.161.104 (talk) 21:33, 9 April 2008 (UTC)[reply]

I compliment you Serge Voronoff for doing something that I don't think any of us would do. I also thank you for being a nut house. You don't get your undies in a bunch. Maybe the person had read the article. They asked "is it true". If I read that article I probably would have been a little skeptical myself if I didn't know for a fact that it was true. Sometimes we can't trust absurd information like that.

Always

Cardinal Raven

71.142.208.226 (talk) 22:53, 9 April 2008 (UTC)Cardinal Raven[reply]

Tequila Slammers

Why does one have salt with a Tequila Slammer? —Preceding unsigned comment added by Quidom (talkcontribs) 20:49, 9 April 2008 (UTC)[reply]

According to our Tequila article, salt lessens the tequila's "burn." bibliomaniac15 Hey you! Stop lazing around and help fix this article instead! 20:52, 9 April 2008 (UTC)[reply]

Hand sign

While driving through Los Angeles, California, USA several months ago, I came across a group of young African-American teenagers. As I drove past them, they flashed this hand sign at me; I am unsure whether if it was merely a sign of acknowledgment or whether it's a gang-related sign or whether it was vulgar. The sign was done with the right hand and consisted of an upside-down "V" (like an upside-down peace sign) with the thumb through at the base of the index and middle finger.

Just by doing some brainstorming, I realized that it sort of looks like a vagina with the thumb for a clitoris. Can someone help me out? Thanks. Acceptable (talk) 22:11, 9 April 2008 (UTC)[reply]

More brainstorming, looks like a guy with a penis. Maybe a vulgar way of saying "you're a cock". Julia Rossi (talk) 22:34, 9 April 2008 (UTC)[reply]
I'd agree with Julia. The hand sign for a vagina is done by pointing out with the index finger and thumb of each hand while the other fingers are folded in, then touching the tip of the index fingers together and touching the tips of the thumbs together at the same time. You'll have sort of a diamond shape. The closer you move your hands together, mashing your digits together, the more it will resemble a vagina. Dismas|(talk) 22:39, 9 April 2008 (UTC)[reply]
I found maybe the cousin here[7] (lowest left) of the fig hand when the fingers are curled back it's vaginal, so the "standing man" could be a newer variation about the cock, and saying "f___ you." Provocative.  :-/ Julia Rossi (talk) 00:24, 10 April 2008 (UTC)[reply]
It is possible that these young teenagers (the OP does not specify male or female or both) are transmitting these genitalia-representing signs as ways of signalling their own gender identity. They may be in adolescent confusion about whether they are masculine or feminine enough, and metaphorically expose their sex organs to strangers as a way of solidifying their identity. They greet others they do not know by saying "I am male!" or "I am female!" as the case may be, to make it clear to the outsider. (A similar use of semiotics to visually signal gender is in the wrapping of newborn babies in pink or blue blankets, although that of course is done on behalf of the individuals in questions, babies not having much autonomy, understanding of their culture's assignation of random colours to gender, ability to clothe themselves, or interest in much besides breasts and sleeping.) These teenagers may need the psychological validation of repeatedly affirming their gender choice in front of their peers; thus we can hypothesise that although the hand display is presented as if to the passing motorist, it is in fact covertly targeted at the surrounding peer group, thus conflating individual self-determination and group solidarity. It is possible. BrainyBabe (talk) 07:48, 10 April 2008 (UTC)[reply]
OR...it was an A. Doing it myself it looks like an A. Perhaps their gang begins with an A, or you were in an area beginning with A. 81.96.161.104 (talk) 17:46, 10 April 2008 (UTC)[reply]
Mmm. As in "at ya"? Comparing ORs, the base means near the hand and the thumb goes through. I can only get an A if the thumb doesn't go through and is nearer the finger tips, say across the middle knuckles. Julia Rossi (talk) 22:18, 10 April 2008 (UTC)[reply]
Yeah, but if you bring the thumb down slightly it creates a little gap, it wouldn't be visible from a car as you drove past but if you were closer it would clearly be an A. Not to knock the soul-searching and introspective nature of the average gang-member, but it seems far more likely that they were signing a letter than affirming their gender identity. The A serves as a mark of distinction, seperating them from the gang round the corner, making members visible to the trained eye. It could be a vulgar symbol like you said, as in "you're a dick", but it just seems that in the 'gang' context a letter is more likely. Atlanta gangs/Dirty South rappers use a similar sign as an A, just sans the thumb. Although the wikipedia page doesn't have any gangs beginning with A that I could see. But I'd have thought that if it was a 'swear word' gesture then it would be slightly better known. You can't offend someone if they don't know you're being rude. I really want to know what this sign was about now. 81.96.161.104 (talk) 22:30, 10 April 2008 (UTC)[reply]
Right, I've got it. The bottom left of this page explains it's the sign for the Hoover Crips, who are redlinked from HGC on wikpedia as the Hoover Gangster Crips. 81.96.161.104 (talk) 22:35, 10 April 2008 (UTC)[reply]
You could also consider the possibility that it was totally meaningless. Djk3 (talk) 22:42, 10 April 2008 (UTC)[reply]
Ha. The barnstar of superb infra-detection goes to you, 81.96. The Hoover Crips sign seems the most resonating: offensive, ballsy, and brand wise. *am now making the horned handsigns of respect across the chest* Word, Julia Rossi (talk) 23:02, 10 April 2008 (UTC)[reply]
Ha ha thank you. In the immortal word of Ali G: Respec'. 81.96.161.104 (talk) 23:44, 10 April 2008 (UTC)[reply]
Ha triple ha. The sex-obsessed refdesk editors get trumped by the literacy-signalling youth. Of course, you do realise that the "A" handsign only puports to represent a gang. It is possible that these youngsters are really informing the passing motorist of their latest grades at school. It is a visual boast of academic success. BrainyBabe (talk) 06:14, 11 April 2008 (UTC)[reply]
and this means Acceptable that next time it happens, make the peace sign (checking first that it isn't co-opted by another gang) as you jump out of your car, running low in case of drive-bys and asking for their autographs. You've just been signed by legends!!  ; ) Julia Rossi (talk) 09:16, 11 April 2008 (UTC)[reply]

Haha, thanks guys for the response! Yeah the link posted by 81.96 shows the correct hand sign that I saw. Any idea where Hoover is? Acceptable (talk) 02:22, 12 April 2008 (UTC)[reply]

Our article Crips shows they are 30,000 strong with plenty of infighting all over & LA. Dey impreziv. As for the Hoovers, they found you, brah! Do yo homework an' good luck fo nex' time! (no disrespec') Julia Rossi (talk) 09:21, 12 April 2008 (UTC)[reply]


April 10

Niiko

Moved to language desk.

WP:MoS editors looking for places to point people for help with page layout and graphic design

Hi, we get questions at WP:MoS from time to time along the lines of what kinds of borders to use around pictures, how to arrange the elements of a page to meet good desktop publishing and graphic design principles, etc. Can you guys recommend any links? - Dan Dank55 (talk) 04:06, 10 April 2008 (UTC)[reply]

P.S. I searched the 100 links I got from a Google search on the Ref Desk/Computer archives on "DTP" and "Image Layout", no luck. - Dan Dank55 (talk) 04:22, 10 April 2008 (UTC)[reply]
Do you mean other than layout in wikipedia? Otherwise there's a google search here[8], and nothing quite like looking at lots of examples of magazine layouts (glossy, newsprint, websites) analysing and imitating them. Julia Rossi (talk) 05:27, 10 April 2008 (UTC)[reply]
Yep. I agree with your approach, but the question keeps coming up, I'll try the Google search. This was my reply in WP:MoS:

Yes, hopefully we'll write something up when the current discussions are finished. In a nutshell, the main problem with sandwiching text between images is that not everyone is using the full width of the screen, or using a high resolution, or even necessarily reading Wikipedia on a computer, so you can get some very tightly squeezed text on some screens if you put pictures on both sides. Anyway, I tried to find more information for you on colors or borders, but other than the text and links in WP:MOS#Images, I'm not finding anything on principles of page layout and design, in WP:PICTURE, or the how-to table articles, or in the HELP space, or in WP:LAYOUT, or WP:IMAGE. Can anyone tell me where to find more advice ? - Dan Dank55 (talk) 02:33, 10 April 2008 (UTC)

Eureka. The best site[9] gives you heaps. Ignore the (ironically) clunky layout, and that it's meant for a wide screen, the content (range and all on topic) is good. This example "Getting Started with Page Layout"[10] supports your answer, from "Less is more" onwards. Googling "graphic design principles" gives you others to explore; but this is the best for starters. Julia Rossi (talk) 13:48, 10 April 2008 (UTC)[reply]
Thanks loads, I'll copy your answer to WT:MoS. - Dan Dank55 (talk) 18:59, 10 April 2008 (UTC)[reply]

Werewolves Clear them Up

Could someone clear up werewolves for me? When a werewolf transform during the full moon I remember in a book reading that they transform and wait for the moon to wane. So that would mean that its like a woman's menstrual cycle once a month for a whole week? Do they transform back into their humans form when the sun comes up? Or do they stay like a werewolf until the moon disappears? 71.142.208.226 (talk) 05:59, 10 April 2008 (UTC)Cardinal Raven[reply]

In all the (modern) stories I've seen, the werewolf transformation is a nocturnal thing - the wolf-man is a man during the day. Whether the change occurs only on the night of the full moon, or for 3 or 5 nights is a matter of more controversy. Steewi (talk) 06:14, 10 April 2008 (UTC)[reply]
Have you read werewolf? Being fictional, they can do whatever the writer wishes them to do.--Shantavira|feed me 09:19, 10 April 2008 (UTC)[reply]

I read the werewolf article. It helped, but not a lot. I was just asking this question cause it still is very confusing. I want to write a good werewolf story. I don't want to make it sound fake or untrue to the werewolf. Lately, stories have said that silver bullets kill vampires. I remember when I was young wart hog when silver bullets killed werewolves not vampires. So I wanted to keep to that mythos. But I still need to know how I'm going to do the transformation. Anything the writer wishes them to. So could it be for a whole week?Cardinal Raven (talk) 15:21, 10 April 2008 (UTC)Cardinal Raven[reply]

Why not make it so they only completely transform on the night of the full moon, and transform to a lesser degree as the moon wanes ? On a night with a crescent moon they could just become irritable and a bit hairy, for example. StuRat (talk) 15:55, 10 April 2008 (UTC)[reply]

Thanks for the idea Sturat. Sounds good. Thanks for all the help.71.142.208.226 (talk) 17:14, 10 April 2008 (UTC)Cardinal Raven[reply]

You might also consider adding the element of Clinical lycanthropy to your story.--Lenticel (talk) 21:51, 10 April 2008 (UTC)[reply]

Car brand perception

My wife has a Fiat car, which is pretty much a budget car in the UK. She is American and she told me that when she tells her friends in the US that she has a Fiat they will be impressed. On the other hand Volkswagen is seen as a quality brand in the UK, but a budget brand in the USA. The pricing follows the perception. Do the car manufacturers deliberately market their cars at different market segments in different countries, or is this something that just happened? -- Q Chris (talk) 07:21, 10 April 2008 (UTC)[reply]

All "original research" here, but I don't know of anybody here in the colonies that would be impressed by a Fiat, except perhaps somebody who's stuck with a Yugo. In fact, I have heard it said that Fiat stands for "Fix it again, Tony." Volkswagen probably does still carry the budget image, going way back to the days of the original Beetle. --LarryMac | Talk 13:10, 10 April 2008 (UTC)[reply]
Some seemingly very new poster ads for the French Citroen cars here in the UK are designed to drop huge hints that they're German in origin, or Germanic in quality. I doubt their direct equivalents will be used much on mainland Europe. And IMHO, the VW is very much a middle-of-the-road motor, no pun intended, and not exactly a quality marque. Theediscerning (talk) 15:12, 10 April 2008 (UTC)[reply]
If you're looking for a better example, Mercedes has a distinct luxury image in the US, where they only sell upmarket cars. However, I understand that in Germany they sell a full range of vehicles, including entry level vehicles and commercial trucks. VW, on the other hand, does not have a luxury image. Their upmarket brand, Audi, is a bit better. As was said previously, a Fiat would impress nobody, although the Montreal cab driver who actually got airborn driving me over the river in a Fiat certainly managed to get my attention. Unfortunately for your wife, simply declaring a Fiat to be impressive does not make it so. StuRat (talk) 15:51, 10 April 2008 (UTC)[reply]
Here in the states, I'd expect a Fiat to be a small sports car. So, people that like small sports cars might be impressed by it. I'm guessing many of the "normal" cars Fiat makes would not be legal to sell here, so the brand isn't widely seen. Friday (talk) 15:57, 10 April 2008 (UTC)[reply]
The Fiats I've seen in the US were in no way sports cars, they were econoboxes. StuRat (talk) 16:03, 10 April 2008 (UTC)[reply]
You can't get the Fiat X1/9 in the States? DJ Clayworth (talk) 16:06, 10 April 2008 (UTC)[reply]
Yes, and before that, the Fiat 124 Sport Spider was also available in the US. Both were definitely "sports cars" in my )(and my friends') book. I knew a fellow who very much like his Spider.
Atlant (talk) 17:19, 10 April 2008 (UTC)[reply]
I'm guessing, I've only looked closely at the ones that looked interesting to me. Maybe I've seen others, and not known they were Fiats, if they looked like random econoboxes. I can't remember the last time I saw a Fiat (that I knew of) on the street. But, yes, when I've seen them, they've been little sports cars like the one Clayworth linked to. (Altho, if I saw that, I might think it was a Triumph or something.) Friday (talk) 16:10, 10 April 2008 (UTC)[reply]
According to this website Fiat left the US market in 1985. Yes, the X1/9 was available here - there's even a caption in that article indicating such, but the quality was suspect (also mentioned in that article, albeit without sources). On this page, the first comment posted to the story reveals the perceived quality issues. I honestly can't recall what other models might have been sold here. --LarryMac | Talk 16:24, 10 April 2008 (UTC)[reply]
Re the sporty fiat, Seinfeld's 1967 Fiat "BTM" coupe brought attention in a prang. Julia Rossi (talk) 08:43, 11 April 2008 (UTC)[reply]

Bigger than Texas?!

There have been increasing references recently on eco sites to some sort of huge island of plastic debris circling in the Pacific Ocean. The references all seem to contain similar phrases which might indicate a spreading urban myth. Can anyone point me to a site that has photos of this alleged phenomenon. Richard Avery (talk) 07:49, 10 April 2008 (UTC)[reply]

Hi Richard, the oceanographers and flotsam boffins are onto it(eg here[11] in legit sources like the BBC,The Independent, news.com.au and google images[12] bear it out. One place says it's bigger than the US! scary stuff (the media I mean) Julia Rossi (talk) 08:03, 10 April 2008 (UTC)[reply]
See also: Great Pacific Garbage Patch. --antilivedT | C | G 08:23, 10 April 2008 (UTC)[reply]
Yuk. Can we see this gunge on Google Earth? --Richardrj talk email 08:32, 10 April 2008 (UTC)[reply]
They are mostly tiny bits of plastic, and with a mean density of 5.1kg/km² (according to our article) I would expect not.--Shantavira|feed me 09:31, 10 April 2008 (UTC)[reply]
The flotsam experts also speak in confetti terms, but theory fails the eyeball test with this plastic high tide[13] and this[14] captioned "The Pacific Ocean trash vortex contains more waste than the largest landfill on earth" from the article[15]. Julia Rossi (talk) 09:50, 10 April 2008 (UTC)[reply]

Hey, thanks Julia and Shantavira and Antilived and Richardrj - very helpful, it seems it is true. Now we wonder how long before we can walk from California to Japan. Richard Avery (talk) 15:21, 10 April 2008 (UTC)[reply]

Hi. Note that Google Earth usually shows only the sea floor and no arctic ice cap when viewing ocean. Thanks. ~AH1(TCU) 01:47, 13 April 2008 (UTC)[reply]
Google Earth is predicting the future. -mattbuck (Talk) 20:40, 14 April 2008 (UTC)[reply]

Why does Yahoo jilt Microsoft?

I'm compiling a list of probable reasons why, bottom line, Yahoo wants nothing to do with Microsoft. Do you know other reasons why Yahoo may not be interested in merging with Microsoft besides its low purchase offer? So far, I have the following:

  • Microsoft has added functions like transparency to dragged objects but has consistently refused to do anything about many more horrendous shortcomings, like providing a built in automatic duplicates removal tool in Outlook Express and Outlook, leaving such functions to be purchased separately through third parties or Microsoft Marketplace.
  • DRM. Microsoft ignores the fact that digital audio signals are converted to analog audio signals to accommodate human listening and that analog audio signals can be faithfully recorded and reconverted to digital.
  • Internet Explorer is incompatible with many web sites acclimated to Mozilla Firefox although Mozilla Firefox has similar shortcomings at sites developed with Microsoft applications software.

Do you know other reasons besides immediate financial gains or losses why Yahoo has jilted Microsoft?

71.100.160.37 (talk) 13:34, 10 April 2008 (UTC) [reply]

I heard Microsoft jilted Yahoohotclaws 13:43, 10 April 2008 (UTC)[reply]
What else do you expect Microsoft to say? 71.100.160.37 (talk) 14:45, 10 April 2008 (UTC) [reply]
While your listed items are valid reasons for a consumer to avoid doing business with Microsoft, they really have no bearing on a takeover bid. Since it is a publicly traded company, Yahoo's executive officers and directors have a fiduciary duty to their stockholders to obtain the highest price they can. Yahoo thinks the MS offer is less than it should be, and that's really all that can be said. --LarryMac | Talk 14:56, 10 April 2008 (UTC)[reply]
If that were true then a company fronting for China, Iran or for the former Soviet Union could easily offer double the Microsoft price, since price would be the only issue that Yahoo executives would be obligated to consider. 71.100.160.37 (talk) 15:41, 10 April 2008 (UTC) [reply]
There would be other considerations at that point. As an example, look at the history section in the Unocal article. --LarryMac | Talk 15:48, 10 April 2008 (UTC)[reply]
Other considerations indeed, especially when you read the next topic. So back to the list of other considerations, if you will. I would simply like to expand the list to cover issues which Yahoo might not want any part. 71.100.160.37 (talk) 20:10, 10 April 2008 (UTC) [reply]
The business pages would be a good place to look for informed guesses about this sort of thing. Yahoo's objections aren't likely to be the same as yours or my objections (as LarryMac says). You might be interested in this analysis. AlmostReadytoFly (talk) 15:28, 10 April 2008 (UTC)[reply]
Granted, no one's list might be the same as your's or mine, which is my purpose here of developing one that is combined. 71.100.160.37 (talk) 20:18, 10 April 2008 (UTC) [reply]
But the point being made above, I think, is that nothing on your list seems in any way at all to be a plausible reason for Yahoo's reluctance. --Tagishsimon (talk) 21:40, 10 April 2008 (UTC)[reply]
True only if you are in such dire straits that you have no choice but to abandon your philosophy of purpose to survive. Assuming that Yahoo is in dire straits and only unwilling to abandon its philosophy of purpose then my question is what might the basis of its unwillingness be? Is there anything which Microsoft represents that Yahoo wants no part or am I incorrectly assuming that their are living breathing human beings at the helm of Yahoo rather than mechanical gobblee gooks manning the tiller? 71.100.160.37 (talk) 01:26, 11 April 2008 (UTC) [reply]
Many many companies do everything they can to avoid takeover attempts. (See Takeover#Tactics against hostile takeover for an overview of techniques to do this, and poison pill for probably the most popular one.) It is true that the officers and directors have a fiduciary duty to their stockholders to get the best price, but they also have a personal interest in not losing their jobs. Generally this is much of the reason that companies hold out even in the face of reasonable bids. (I have no idea whether this is a good deal in this particular instance.) Much law has centered around the tension between D&Os' duty and their potential to entrench themselves, trying to define just how much a corporation can resist a decent takeover bid--see e.g. Unitrin, Inc. v. American General Corp. and Unocal Corp. v. Mesa Petroleum Co.. Mangostar (talk) 22:36, 10 April 2008 (UTC)[reply]

hi

what are the odds that i will live to 150? i am 11 now. thanks. David. —Preceding unsigned comment added by 79.77.251.12 (talk) 14:23, 10 April 2008 (UTC)[reply]

Hi! We have an article on oldest people. It says that the verifiably oldest person lived to be 122 years and 164 days old, so no one in the past lived to be 150 (if we exclude people in the Bible like Methuselah, whose age may have been exaggerated for story-telling purposes). While it is true that the age of the very oldest people is going up (in 1837 the oldest verified person was 108), it's difficult to speculate what things will be like in 100+ years. -- 128.104.112.85 (talk) 15:04, 10 April 2008 (UTC)[reply]

Starting in 2005 I noticed a serious trend where people were taking their health and diet a lot more seriously. I think there is a health revolution going on. If there were, and in the future we will have people routinely living to 150 and beyond, there would be little or no statistical sign of it now. So just because the average age in the western world is about 73 and the longest-lived person was about 120, that doesn't mean that both average age and maximum age can't be pushed up and up ad infinitum.

And of course your phrasing, 'what are the odds' is a bit naive. Chance has nothing to do with it. Genetics, mental health, geographical location, and the will to live all play huge roles in determining your 'odds' to live to 150. Vranak (talk) 15:37, 10 April 2008 (UTC)[reply]

It's tough. As the above note it is a major increase on the current age. It's always important to note that average-age increases over the past 100 years are said to owe as much to improving child-survival than they are to medical/healthcare advances (in the old days if you survived childhood it wasn't amazingly surprising to live to 70/80). Chance plays a role because...well because you cannot expect to be able to control/predict everything. What if your country suddenly gets involved in a major war, or your local region is hit by a tornado/earthquake, what are the chances of being in an accident etc. etc. Chance plays a role but most 'life' underwriters assess knowing that a large amount of the risk is in your health & lifestyle. With medical advances and future technology i'm hopeful that people of your generation can live for much longer, more active lives than your forefathers, I hope medical science can prolong life but perhaps 150 is too much of a change in too short a time-scale...after all any such technological/medical advances will doubtlessly start off the preserve of the rich, then slowly become more viable to the poor as the technology ages/scale of usage increases. ny156uk (talk) 15:47, 10 April 2008 (UTC)[reply]
Chance plays a role if you aren't clear about which behaviors have a serious detrimental effect on your lifespan. They are of course the main inputs: food, water, and air. Psychosocial inputs are also critical of course. Vranak (talk) 16:10, 10 April 2008 (UTC)[reply]
Based on the number of super old people currently living, compared with the number of people on earth, you're "odds" of living to 113 are about 1 in a billion. Living 30 years past the all-time record? Good luck. Useight (talk) 16:45, 10 April 2008 (UTC)[reply]
By this logic I could argue that the odds of one of those super old people living to be 113 are also one in a billion, while ignoring 'logic' their odds are plainly one in one, as they are alive right now (by definiton). See also destiny. Conclusion: calculating statistical probability is a fruitless endeavour. You're either going to live to 150 or not. Vranak (talk) 17:19, 10 April 2008 (UTC)[reply]
Nonsense. You're comparing two different questions "Given that I am currently 113 years old, what are the odds that I will live to the age of 113?" is obviously 100% "Given that I am currently 11 years old, what are the odds that I will live to the age of 113?" the odds are tremendously lower. The fact that anyone will live a finite and definite lifespan, does not negate the idea of using statistics to figure out what the most likely value for that definite number is.
You're suggesting that all probability is worthless. That is a silly position to take. The universe may have pre-determined that a certain horse will win a race, but unless I can magically see into the future, I need to use probability to figure out which horse to bet on.
Vranak, if you happen to know precisely how long user 79.77.*.* will live, then please simply provide us with the number and simplify this discussion. Otherwise, we need to admit that our data is incomplete and deal in probabilities. APL (talk) 18:58, 10 April 2008 (UTC)[reply]
I will not be drawn into discussions that involve accustions of 'nonsense'. Best regards — Vranak (talk) 23:07, 10 April 2008 (UTC)[reply]
I'll make a note that the next time I feel the need to argue that someone else's argument is nonsensical, I'll use more a obscure way to phrase it. APL (talk) 14:35, 11 April 2008 (UTC)[reply]
I understand that when someone hasn't the wit to understand a point, they'll often resort to name-calling and pejorative labelling -- nonsense and the like. Vranak (talk) 05:12, 12 April 2008 (UTC)[reply]
I understand that when people make nonsensical points, they'll often resort to ad hominem attacks on the other person's intelligence, so as to avoid having to justify, or even explain, their position. APL (talk) 20:14, 12 April 2008 (UTC)[reply]
The average lifespan in the West has increased significantly (from ca. 35 in 1770 to ca. 80 now), but the maximum lifespan has hardly changed at all. That 108-year-old back in 1837 was almost certainly not the oldest person alive; only a small percentage of people back then had birth certificates or could prove how old they were. The maximum age is affected not by the "will to live" but by chance, lifestyle, access to healthcare at all ages, and above all genetics. Lifestyle affects the difference between dying at 40 and dying at 80, not the difference between dying at 100 and dying at 120. There are some studies that show that supercentenarians (those over 110) often have increased amounts of amyloid protein in their blood shortly before death. That may be a clue that death at or around 120 is 'programmed' into us.
Unfortunately, there are those who claw for immortality and who are illogically convinced that their own "will" can somehow save them from the common fate of all of us. Will means nothing if your plane is about to crash, if the driver on the other side of the median is drunk, or if your immune system makes a tiny mistake and allows a few mutated cells to develop into pancreatic cancer. Nothing. Nothing. --NellieBly (talk) 17:31, 10 April 2008 (UTC)[reply]
Lifespan is of course affected by will to live. People with mediocre lives aren't sure that life is worth living, so they hedge their bets by doing unhealthy things that won't kill them instantly but also won't keep them around much past eighty. But to simplify, we could say that will to live is in turn determined by environmental and hereditary factors, so I do see your point. Vranak (talk) 18:52, 10 April 2008 (UTC)[reply]
The idea that 'will to live' affects how long people actually live, beyond the basics of suicidal behaviour, etc, is poisonous and proven false. Too many people peddle the idea that if you have cancer and really want to live, you can fight it. The corolary is, of course, that anyone who dies of cancer must not have wanted to live enough, and therefore it is their fault. Studies (which I should probably find) show that there is no link between 'will to live' or 'keeping a positive attitude' and surviving cancers. If what you actually mean is that cautious people who take care of themselves tend to live longer, that is more likely. Although they may stress themselves to an early grave! 130.88.140.110 (talk) 20:30, 10 April 2008 (UTC)[reply]
Proven false you say? Proof is a subjective matter. You may be wholly satisfied that a certain proposition is false; all your friends and family may also be totally convinced. But no matter can be considered 'proven' ultimately. And that's a good thing -- as soon as a matter is settled it comes back and bites you in the ass.
Furthermore, saying that an idea is 'poisonous' is akin to making an accustion of heresy or witchcraft. If an idea is false it will be sounded rejected, if true it will be retained. There are no poisonous ideas, only poisonous poisons. Example: recycled hospital air. Vranak (talk) 23:22, 10 April 2008 (UTC)[reply]
"No matter can be considered 'proven' ultimately". What? Really? Do you actually believe that? Do you actually follow such high standards of truth that even your own existence is uncertain to you? 130.88 made a valid point that the media-enforced belief that will-to-live can overcome illness is potentially damaging: even if one patient refuses chemo based on their idea that "they can beat this" and subsequently dies, then I think that idea will have done enough to merit the label "poisonous". Since the mid-eighties there has been a large increase in doubt in Western medicine, fuelled by flawed studies about the effects of alternative medicine, the "helaing power" of prayer and the human body's ability to recover. In seems to me very likely that these beliefs, which HAVE been shown to have minimal, if any, effects will have killed at least one person by now. To use alternative medicine as an example: why would it still be alternative if it worked? Every 'alternative' drug that has been shown to work has been snapped up by drug companies and marketed. If hope kept you alive then it'd be on sale in your nearest pharmacy. 81.96.161.104 (talk) 23:57, 10 April 2008 (UTC)[reply]
I'm sure you're right. Vranak (talk) 01:26, 11 April 2008 (UTC)[reply]
I don't know much about the actual research being done now, but hasn't it been suggested that stem-cell research could extend the lives of humans by decades? Completely artificially, of course, but no more than other medicines. 81.96.161.104 (talk) 17:50, 10 April 2008 (UTC)[reply]
Depends. Are you the last of the "Final Five" Cylons? Clarityfiend (talk) 20:02, 10 April 2008 (UTC)[reply]

Hi david. Barring a Civilization-ending catalolypse, you will live forever. See Technological Singularity. -Arch dude (talk) 02:16, 11 April 2008 (UTC)[reply]

Since you are 11, I recommend that you please do the following:
  • Create an account. This will let us talk to you specifically, instead of talking to a number (an IP address) that is shared between you and other users.
  • Discuss your Wikipedia activities with an adult you trust: a parent, a teacher, a sibling, or whoever. If you insist on trusting strangers, contact me on my talk page: I will respect your privacy. -Arch dude (talk) 02:36, 11 April 2008 (UTC)[reply]
But check out the article Quis custodiet ipsos custodes? which, in essence (and with no disrespect meant to the Arch dude) translates as, how to you know whether the Arch dude can be trusted? --Tagishsimon (talk) 11:18, 11 April 2008 (UTC)[reply]
Or better yet David, don't trust anyone over 30. Vranak (talk) 03:58, 11 April 2008 (UTC)[reply]
That could easily be dealt with by eliminating the over 30s Lemon martini (talk) —Preceding comment was added at 11:11, 11 April 2008 (UTC)[reply]

30 and 20.

Written on my vitaminwater bottle, next to the "1 PETE" plastic symbol is the phrase "30 may be the new 20, but green is definitely the new black. Please recycle". I was wondering what "30 may be the new 20" is referring to. Is it some kind of pop culture reference I'm not aware of? 206.252.74.48 (talk) 16:11, 10 April 2008 (UTC)[reply]

That would be a snowclone. Most commonly when ages are placed into the phrase, they are a bit higher - e.g. "Fifty is the new thirty" - meaning people who are fifty years of age are not "over the hill". But marketing types can and will manipulate a phrase in any way they see fit. --LarryMac | Talk 16:29, 10 April 2008 (UTC)[reply]
For those who don't want to click through, a snowclone is a type of wordplay exemplified by X is the new black. BrainyBabe (talk) 21:16, 10 April 2008 (UTC)[reply]
For many many examples of "X is the new Y" formulations, see http://www.thediagram.com/6_3/leisurearts.html and http://thisisthenewthat.blogspot.com/ . Mangostar (talk) 22:40, 10 April 2008 (UTC)[reply]
Thanks, I'm all clicked out. Only a matter of time before marketing types made snowclone the new buzzword. Typical. Julia Rossi (talk) 11:24, 11 April 2008 (UTC)[reply]
As suggested, the '30 is the new 20' quote would be used to indicate that essentially you're just starting on your adult life at 30 these days, as you at once would have been at 20, and are still young and hip at that age. But why use that on the bottle? Undoubtedly the makers and marketers of this product have done extensive research to find that their most likely market for this particular waste of money are those in that around-30 age range. The implication being, 'buy this drink and you're still part of the cool young group'. --jjron (talk) 17:08, 11 April 2008 (UTC)[reply]
Eggcorn is the new snowclone. BrainyBabe (talk) 19:07, 11 April 2008 (UTC)[reply]
Eggzackly! So Jjron's catch and BB's, makes for a snowjob leading to an eggclone. Julia Rossi (talk) 23:15, 11 April 2008 (UTC)[reply]

Biscuits/cakes

Reading the table at the bottom here, I was startled by the caramel-and-chocolate covered vs chocolate covered shortbread. Out of interest, does anyone know if caramel digestives are charged the same VAT as chocolate digestives? (digestive biscuit for those external to the UK) 130.88.140.121 (talk) 17:03, 10 April 2008 (UTC)[reply]

I'd opine yes, since it is cakes & breads & suchlike which do not attract VAT. From the BBC news: "Under UK tax rules, most traditional bakery products such as bread, cakes, flapjacks and Jaffa Cakes are free of VAT, but the tax is payable on cereal bars, shortbread and partly-coated or wholly-coated biscuits." [16] --Tagishsimon (talk) 17:07, 10 April 2008 (UTC)[reply]
Jaffa cakes explains the legal ruling that ensured they remain zero-rated for VAT. BrainyBabe (talk) 21:12, 10 April 2008 (UTC)[reply]
Jahr, das ist gut but a shortbread bar partly coated in caramel and then chocolate is VAT free, while a shortbread bar without coating or partly coated in chocolate is charged VAT. This was the puzzling thing about the table. So I was wondering about the situation when the base was digestive biscuit, not shortbread. I suppose I really need someone to check a recent receipt from when they last bought both, to see if they have the little VAT mark. 130.88.140.4 (talk) 22:39, 10 April 2008 (UTC)[reply]
There being doubt and uncertainty about the shortbread situation does not affect the digestive situation. It's difficult to understand the distinction the beeb alleges the vatman makes about millionaires shortbread. I'm guessing he's partial. --Tagishsimon (talk) 23:16, 10 April 2008 (UTC)[reply]
Hmmm, I suppose there must be some privileges to being a taxman. Declaring that your favourite snack is not a luxury, owing to the addition of caramel, must be one of them. Quite astounding when you consider that sanitary towels still have VAT! 130.88.140.4 (talk) 00:29, 11 April 2008 (UTC)[reply]

Willesden structure

I'm watching the fourth series of The Last Detective, which is set in Willesden. Several times I've noticed, in the background, a structure that might be a stadium, with an oblique arch over it. I'll bet one of you knows what it is! —Tamfang (talk) 23:00, 10 April 2008 (UTC)[reply]

Wembley Stadium? --Tagishsimon (talk) 23:11, 10 April 2008 (UTC)[reply]
I can't be sure because of the angles of the shots, but yeah, thanks. —Tamfang (talk) 00:54, 11 April 2008 (UTC)[reply]
Here [17] is the Stadium from a different angle showing the angled arch. Richard Avery (talk) 06:30, 11 April 2008 (UTC)[reply]

April 11

To Whelm

It is possible to be overwhelmed or underwhelmed. Can you ever just be whelmed?

Yes. The simple and past participle of whelm is whelmed. -Gwguffey (talk) 01:06, 11 April 2008 (UTC)[reply]
But that is a totally different thing to the concepts of over/underwhelmed. Whelmed means "to be covered in water". So can one be whelmed in the sense that one is neither under- nor over- whelmed? In that it would be synonymous with "moderately satisfied"? 81.96.161.104 (talk) 01:31, 11 April 2008 (UTC)[reply]
Your original question simply asked if you could be whelmed, not whether you could be in a state of neither overwhelmed nor underwhelmed? ;) I agree with JackofOz below, I would like to know Arch dude's description of being whelmed. He has me concerned. If being in a state of "over" or "under" is notable, then it seems you could make a case that the normal state would be just "whelmed"....but if you use that particular word, people may just think that you are all wet covered with water. -Gwguffey (talk) 03:17, 11 April 2008 (UTC)[reply]

This appears to me to be completely subjective. I am whelmed by this question, so (on a sample size of one) yes, a person can be whelmed in the sense of your question. -Arch dude (talk) 02:05, 11 April 2008 (UTC)[reply]

Just to clarify, the original question isn't mine, although I believe it was in some teen film I've seen at some point. I think a cheerleader asked it. 81.96.161.104 (talk) 02:15, 11 April 2008 (UTC)[reply]
Arch dude, can you describe the experience of being whelmed, so that we can all look out for it? Thanks. -- JackofOz (talk) 02:33, 11 April 2008 (UTC)[reply]
The quote comes from Ten Things I Hate About You (one of the more watchable teen rom-coms I've seen). The reply in the film (from an equally airheaded teen) is, "I think you can in Europe." Steewi (talk) 03:43, 11 April 2008 (UTC)[reply]

Allow me to pour some cold water on this debate. Whelm is given in the OED as capsize or overturn. Whelmed, then, is capsized. Overwhelm is derived from whelm and means to be turned over, and has been used since c.1400. Underwhelm is a much more modern c.1956 word, rather as cluefull is to clueless .... an ironic sort of a neologism. So one can be whelmed in exactly the same sense as one can be overwhelmed, albeit the first of those words is not very much in common use. --Tagishsimon (talk) 08:41, 11 April 2008 (UTC)[reply]

Words created in this fashion, because people feel there ought to be a parallel construction to a word that is already in use, are called backformations. BrainyBabe (talk) 19:10, 11 April 2008 (UTC)[reply]
Yes. I thought that everybody knew that "underwhelmed" was a backfackformation, so I assumed that the OP was using "whelmed" as a backformation that splits the difference. The question did not strike me as stunning in any dimension, so I was not overwhelmed. It was not so inane that I was underwhelmed, Therefore, I remained in the neutral state between them: whelmed. I am gruntled by the interest in my response. -Arch dude (talk) 23:39, 11 April 2008 (UTC)[reply]
This discussion is quite ane. -- JackofOz (talk) 05:42, 12 April 2008 (UTC)[reply]
It is not wise on Wikipedia, where our writings go into the archive cloud for who knows how long, to assume that "everybody knows" a certain fact. Refdesks are archived and need to be explicit in their helpfulness. I hope this is an ept description. BrainyBabe (talk) 05:54, 12 April 2008 (UTC)[reply]
Several of the people responding to this question are nocent of misusing the English language for their own amusement. StuRat (talk) 13:19, 12 April 2008 (UTC)[reply]
Apologies for rendering this thread tact, but I feel the need to link to the article on unpaired words where you will find a thinkable amount of examples and links with countful more. ---Sluzzelin talk 13:49, 12 April 2008 (UTC)[reply]

Now the question is, can one be kempt? Pfly (talk) 17:37, 13 April 2008 (UTC)[reply]

Yes. That's one I actually use in everyday conversation. Perhaps we should be ruth and let this quietly die... 79.66.105.94 (talk) 01:05, 14 April 2008 (UTC)[reply]

To Whelp

If whelping is giving birth to puppies, what do you call giving birth to kittens ? StuRat (talk) 05:34, 12 April 2008 (UTC)[reply]

"Queening" apparently.[18] which makes the mother Queen for a Day. Σ:-) Julia Rossi (talk) 09:06, 12 April 2008 (UTC)[reply]
Good to know. So, is this because real queens are always "having kittens" ? :-) StuRat (talk) 13:24, 12 April 2008 (UTC)[reply]
Tom and queen are the names given to male and female cats. I have no idea why. SaundersW (talk) 16:06, 12 April 2008 (UTC)[reply]
The names given to un-neutered male and female cats :) 79.66.105.94 (talk) 00:56, 14 April 2008 (UTC)[reply]

Funny question, but curious since my wife said something about it

For some odd reason my wife and me got into a discussion on wiping ones behind. So the question in this case is: Wipe it while still sitting down, or wipe it standing up? --Nick910 (talk) 01:54, 11 April 2008 (UTC)[reply]

How could you wipe standing up? Surely that wouldn't work? 81.96.161.104 (talk) 02:15, 11 April 2008 (UTC)[reply]
Technically if you bend over while standing up, i don't see any reason why you wouldn't be able to --Nick910 (talk) 02:19, 11 April 2008 (UTC)[reply]
Both are quite possible. There are multiple issues involved, including hygiene, flexibility, space available and personal preference. I don't know of any particular surveys on the topic. If it's anything like the toilet roll controversy, it could lead to internet wars. Steewi (talk) 03:35, 11 April 2008 (UTC)[reply]
What toilet roll controversy? --Richardrj talk email 09:41, 11 April 2008 (UTC)[reply]
"Over or Under" is the major controversy. "Crumple or fold" is the minor one. (I think.) And now we have "Sitting or standing". ---Sluzzelin talk 10:17, 11 April 2008 (UTC)[reply]
I'd have thought there's a small possibility that direction of travel might also be in play. Where is the survey at http://howdoyouwipeyourass.com ? The intertubes are letting us down. --Tagishsimon (talk) 10:21, 11 April 2008 (UTC)[reply]
There is absolutely no controversy there. It's "over", obviously. --Richardrj talk email 10:40, 11 April 2008 (UTC)[reply]
Touché. and very dryJulia Rossi (talk) 11:30, 11 April 2008 (UTC)[reply]

But what about the three seashells? And some folks avoid the "which way to roll?" debate by mounting the toilet paper roll with its axis being vertical.

Atlant (talk) 12:38, 11 April 2008 (UTC)[reply]

Which way do they mount it? With the loose end wrapped towards the toilet or away from it? APL (talk) 14:10, 11 April 2008 (UTC)[reply]


As long as you go "front to back" it shouldn't matter.And why has no-one brought up using a swan's neck? hotclaws 14:16, 11 April 2008 (UTC)[reply]

A swan's neck? What? Am I missing something here? Michael Clarke, Esq. (talk) 00:53, 12 April 2008 (UTC)[reply]
Perhaps a reference to Rabelais, Gargantua and Pantagruel, Chapter 1.XIII "How Gargantua's wonderful understanding became known to his father Grangousier, by the invention of a torchecul or wipebreech." (translation by Sir Thomas Urquhart of Cromarty and Peter Anthony Motteux) :
"Afterwards I wiped my tail with a hen, with a cock, with a pullet, with a calf's skin, with a hare, with a pigeon, with a cormorant, with an attorney's bag, with a montero, with a coif, with a falconer's lure. But, to conclude, I say and maintain, that of all torcheculs, arsewisps, bumfodders, tail-napkins, bunghole cleansers, and wipe-breeches, there is none in the world comparable to the neck of a goose, that is well downed, if you hold her head betwixt your legs. And believe me therein upon mine honour, for you will thereby feel in your nockhole a most wonderful pleasure, both in regard of the softness of the said down and of the temporate heat of the goose, which is easily communicated to the bum-gut and the rest of the inwards, in so far as to come even to the regions of the heart and brains. And think not that the felicity of the heroes and demigods in the Elysian fields consisteth either in their asphodel, ambrosia, or nectar, as our old women here used to say; but in this, according to my judgment, that they wipe their tails with the neck of a goose, holding her head betwixt their legs, and such is the opinion of Master John of Scotland, alias Scotus."
Emir Kusturica visualized a variation in Black Cat, White Cat : Dadan falls into an outhouse's steaming pile, and uses a snow white goose to clean himself (It was a set-up involving drinks spiked with laxatives). Ending on a more mannered note, I recommend the article on European toilet paper holder. ---Sluzzelin talk 01:22, 12 April 2008 (UTC)[reply]

Some people do not wipe. When I lived in a dorm in university, there were some Muslim students, and there was a watering can in the bathroom. It took me awhile to figure it out! Adam Bishop (talk) 17:55, 11 April 2008 (UTC)[reply]

See Islamic hygienical jurisprudence, although note that washing, as opposed to wiping, is not restricted to Muslims. BrainyBabe (talk) 19:13, 11 April 2008 (UTC)[reply]
On the over or under debate - over, at least if you have an artexed wall. :( -mattbuck (Talk) 23:45, 11 April 2008 (UTC)[reply]

Why wipe when you have this? --antilivedT | C | G 09:48, 12 April 2008 (UTC)[reply]

Why are your locust getting it on? Don't you think who ever took the picture would give them some privacy.71.142.208.226 (talk) 02:52, 11 April 2008 (UTC)Cardinal Raven[reply]

See privacy. Since they effectively lack self-awareness, most animals don't give a damn about privacy. (Except domestic cats perhaps.)--Shantavira|feed me 08:26, 11 April 2008 (UTC)[reply]
Funny, when I looked at my user page, I wondered the same – I felt like an intruder on my own userpage! The animals who most don't give a damn about privacy are humans with cameras, followed by my neighbourhood's domestic cats. *groan* Julia Rossi (talk) 11:34, 11 April 2008 (UTC)[reply]

Can't you see the two locust are trying to hide in the trees, but that dirty camera man found them. I wonder what the locust are thinking right now?71.142.208.226 (talk) 18:12, 11 April 2008 (UTC)Cardinal Raven[reply]

"Was it a blinding flash for you? It was for me!" Julia Rossi (talk) 23:24, 11 April 2008 (UTC)[reply]

No fly zone?

I was reading the article on the no-fly zone and was doing some thinking. The Iraqi no-fly zones were declared by Britain, the United States and France according to the article on the Iraqi no-fly zone. But can the United Nations declare a no-fly zone over a country or region of a country? Or as part of a Security Council resolution? --Blue387 (talk) 04:18, 11 April 2008 (UTC)[reply]

I don't think so, but it's probably a matter of dispute. In any case the security council didn't pass any resolution on the matter. Algebraist 12:49, 11 April 2008 (UTC)[reply]

departments of wilson airport

How many departments are there in Wilson Airport(Kenya),which ones are they and how does each one of them contribute to the smooth functioning of the airport? —Preceding unsigned comment added by 41.206.49.82 (talk) 08:57, 11 April 2008 (UTC)[reply]

Wouldn't it be nice if posters at least said 'please' !--Artjo (talk) 09:32, 11 April 2008 (UTC)[reply]
And if they signed, gave you a context, a link, googled a bit first, or... I was in a bank when a teller lost it and wouldn't help the customer until he heard the magic word! The fight went on for some time, Julia Rossi (talk) 11:45, 11 April 2008 (UTC)[reply]

Morse Code SMS notification

I suppose many people have noticed that the common SMS notification signal from Nokia phones (... -- ...) translates to SMS. However, I frequently hear another notification signal (I think also from Nokia phones) that goes ... ...- .- which translates to SVA. What's the meaning of SVA in that context? Google searches for "sva nokia", "sva cellphone", "sva sms" etc. have turned up nothing useful so far. -- Ferkelparade π 10:53, 11 April 2008 (UTC)[reply]

Although the original tone they used may have been the morse for SMS, it's likely that now, with the advent of personalised ringtones and the general customisability of mobile phones, that the tone you heard was meaningless. I have to say that I'd never noticed the SMS thing until you pointed it out though, so thanks for the cool bit of trivia! Michael Clarke, Esq. (talk) 01:41, 12 April 2008 (UTC)[reply]

Housing the world

Years ago in high school a geography teacher told me that the entire world could be housed in the state of Texas, if everyone lived in 4 person houses, with enough room for everyone to have a garden, too. Is this true? If it's not, what's the smallest area that the entire world could be housed in? Thanks 81.96.161.104 (talk) 12:35, 11 April 2008 (UTC)[reply]

Texas has an area of 268820 square miles, and the world population is about 6.6 billion, so if everyone lived in Texas, there would be about 105 square metres per person, or 420 per four-person house. That's easily enough space. Algebraist 12:46, 11 April 2008 (UTC)[reply]
Your user name is appropriate! Jørgen (talk) 13:10, 11 April 2008 (UTC)[reply]
For those not entirely fluent in both metric and U.S. measurements, 268,820 square miles is about 696 240 square kilometers or 6.96x1011 square meters. At 6.6 billion people, that cooks down to 105 square meters per person or 420 square meters per four-person household. In U.S. measure, 268,820 square miles is 7.49x1012 square feet; that's 1135 square feet per person or about 4500 square feet per household.
That's a square patch of land about 20 meters or 65 feet on a side—bigger than subdivision lots in many western countries. (A bit of quick Googling finds minimum residential lot size requirements anywhere from 3000 to 4500 square feet in various communities throughout Texas.) TenOfAllTrades(talk) 13:29, 11 April 2008 (UTC)[reply]
Why don't we use Alaska instead? We'd have twice as much space. Useight (talk) 15:48, 11 April 2008 (UTC)[reply]
A lot of Alaska is either mountainous or ice-covered, thus reducing the available space. That is another factor that would need to be considered before we all put on our hats and saddle up for the ride to Texas - not all of the 268820 sq mi / 696240 sq km would actually be usable for living space. And then there's the space required for other things like power plants, sewage treatment, police and fire departments, etc. Teachers say a lot of things that don't bear close scrutiny. --LarryMac | Talk 16:32, 11 April 2008 (UTC)[reply]
You might fit 'em in, but imagine the traffic! --jjron (talk) 16:47, 11 April 2008 (UTC)[reply]
Such utilities can, with enough ingenuity, be built on top of the 4-person houses. However, I don't relish the thought of living inside one of the support pillars for a huge sewage treatment plant. 206.252.74.48 (talk) 18:36, 11 April 2008 (UTC)[reply]
Perhaps building them underneath the houses would be more practical... Thanks for the answers though guys, I still can't really believe it. 81.96.161.104 (talk) 19:04, 11 April 2008 (UTC)[reply]
Also, thanks again for the answers, I'm mostly writing this to let people know that 81.96 is now Michael Clarke, Esq. (talk) 20:30, 11 April 2008 (UTC)[reply]
A big issue would be water supply. A lot of Texas is quite arid. Many of those gardens would not be able to grow much. I suppose the water supply issue could be resolved by lining the Gulf coast with desalination plants. The gardens could get quite small after you deduct the space required for roads, services, and infrastructure. It would end up more or less an urban expanse. Marco polo (talk) 20:30, 11 April 2008 (UTC)[reply]

On a similar line it is said that the entire world population times 10 could be fitted into Loch Ness, Scotland (if it were emptied of water of course!).--Artjo (talk) 07:34, 12 April 2008 (UTC)[reply]

World population 6.6x109. Volume of Loch Ness 7.4x109 m3. Average volume of human body say 0.1 m3. So, yes, that is approximately correct. Would be very crowded though. Gandalf61 (talk) 09:02, 12 April 2008 (UTC)[reply]
We can drain Loch Ness by piping all the water to Texas. Gwinva (talk) 09:39, 12 April 2008 (UTC)[reply]
But Texas wouldn't need the water anymore because all the people are now piled up in Loch Ness. --jjron (talk) 16:02, 12 April 2008 (UTC)[reply]
It may be possible for an intelligent species to evolve in Texas after the disappearance of homo (smirk, lol, rofl) sapiens. For which dihydrogen monoxide is a prerequisite. --Cookatoo.ergo.ZooM (talk) 22:45, 12 April 2008 (UTC)[reply]
Hi. Well, I also heard that you can fit the entire world's population in Virginia. Also, I heard that you can fit (although for practical reasons this would not be good as everyone would be squished under one another and have no space) in a box, 1 km x 1 km x 1 km, and this box would fit in the Grand Canyon. Hope this helps. Thanks. ~AH1(TCU) 01:37, 13 April 2008 (UTC)[reply]

In summary, the world's population could live in Texas, but wisely chooses not to. Pfly (talk) 08:58, 13 April 2008 (UTC)[reply]

The UK version of this is that the population of the world could stand on the Isle of Wight. Which probably isn't true anymore as the world's population is bigger than it used to be and the IoW is smaller than it used to be. -88.109.180.121 (talk) 09:02, 13 April 2008 (UTC)[reply]

You wouldn't need anything as big as Texas if you could stand the crowding. Matt Deres (talk) 16:59, 14 April 2008 (UTC)[reply]

E numbers

I have perused our article on various E number, such as E420, E331 ect, but find it a bit uninformative. What I wish to know is why are these put into foods and drink? What are they? for example, E472 (if I remember clearly)is an emulisifier, or is able to cause oil and water to mix. But why is this put into my bread (UK Kingsmill) Surely bread should be a time tested recipe of wheat, flour, eggs, salt ect. thousands of years old. Why are these things put into almost every food stuff, includeing steak, bacon, redbull, bread, canned fruit. Has anyone tested these for cancer causing agents, ect, are these healthy? any relevant info would be greatly appreciated. Why is there so little info on these? who decided to add these to whatever? thanks, and stay healthy. —Preceding unsigned comment added by 86.18.34.51 (talk) 17:52, 11 April 2008 (UTC)[reply]

Most of the time I'd imagine it's a financial decision. If E120 is cheaper than the "natural" ingredient, it makes fiscal sense to use it. Some of the time it may be that people are actually used to the taste of the E numbers and so would rather have E-strawberry than real-strawberry. About the testing, as far as I know the problem is that many of them were not tested adequately when they were approved for human consumption and much of the data coming out now, while hardly damning, is indicating that there are probably side-effects, especailly among children. This article from a few days ago mentions that some e-numbers have been linked to aggression in children. I'm not sure but I think that much of the problem is that the results are still very debatable. 81.96.161.104 (talk) 18:42, 11 April 2008 (UTC)[reply]
The book Twinkie Deconstructed (ISBN 1594630186) explains how all of the ingredients in a Twinkie are made, and what their role is. Since it's a U.S.-centric book, it doesn't talk about E numbers, and many of the manufacturing choices are driven by government policy (subsidies, tariffs, etc.), but it will provide some insight into what drives the decisions. There are lots of other recent books concerning modern agriculture and diet (Omnivore's Dilemma, What to Eat, etc.) that may also interest you. -- Coneslayer (talk) 19:14, 11 April 2008 (UTC)[reply]
In the case of bread, additives may be used to adjust the texture (my home-made bread is much denser at the bottom than the top), condition the crust (mine develops a hard, chewy crust if it's not cooled properly), inhibit mold (mine will start to grow moldy in less than a week), and other things. --Carnildo (talk) 22:03, 11 April 2008 (UTC)[reply]
As Carnildo says, emulsifiers are used in bread as dough strengtheners and crumb softeners. (In fact, I believe that's why the eggs are included in your "time tested recipe".) In commercial bakeries, they stabilize the dough during processing. Various emulsifiers are examined in this article. - Nunh-huh 03:19, 12 April 2008 (UTC)[reply]
And they can't just make the bread with traditional recipes because you want your bread homogenous, fresh and long lasting. If you want fresh bread, and it's being mass produced, it's going to have to be treated somehow to keep it fresh long enough to make it to the shelf from the bakery/factory. You then want it to last for days without getting mouldy or stale, so that means more treatment. And you want every loaf the same, which means traditional methods aren't going to cut it. Also, bear in mind E-numbers can be used for fairly ordinary things like salt. It's actually a pretty common 'trick' to write out the full name of all your ingredients rather than putting the E-number, as people assume 'real names' are more healthy. E-numbers are just catalogue numbers for the permitted food additives in the EU; being an E-number or not is not what makes something problematic. 79.66.105.94 (talk) 00:51, 14 April 2008 (UTC)[reply]

Tibetan Olympians

After reading this article about the IOC's stand on political protest during the Olympics, I wondered if there were any Tibetan Olympians. Competing for China, or of Tibetan origin but competing for another country. Anyone know? 81.96.161.104 (talk) 18:34, 11 April 2008 (UTC)[reply]

What is the legal minimum age for buying and drinking alcohol in Las Vegas Casinos, and for gambling in them too? Thanks. 92.9.155.232 (talk) 22:08, 11 April 2008 (UTC)[reply]

21 192.45.72.26 (talk) 22:23, 11 April 2008 (UTC)[reply]
I'm pretty sure 21 is standard everywhere in the US (except Indian reservations, I guess). Useight (talk) 22:27, 11 April 2008 (UTC)[reply]
Thanks for the above answers. BUT - would a nearly 21 year old and mature looking UK guy be asked to prove his age on entering the casino? —Preceding unsigned comment added by 92.9.147.1 (talk) 10:43, 12 April 2008 (UTC)[reply]
Depends on the casino and who's on the other side of the conversation. Most times that I buy alcohol here in Vermont, I get carded. And I'm in my 30s. Most states have laws stating that "everyone" has to be carded because having the cashier "estimate" or "guess" that the person is old enough is not good enough. Dismas|(talk) 13:09, 12 April 2008 (UTC)[reply]
At one time I worked as a mystery shopper and checked discount gas station chains for compliance. Since the cashiers always had a line the companies relied on the legal penalty for the cashier rather than its own policy of complying with the law. My job was to find people who were 21 or older but who looked younger as my rider. In all but 2 cases out of hundreds did the cashiers fail to ask for ID. In both cases the cashiers were newly hired and not well enough trained. 71.100.160.37 (talk) 17:10, 12 April 2008 (UTC) [reply]
I'm 24 and I get carded buying M-rated video games. And you only have to be 17 to buy those. Useight (talk) 17:59, 12 April 2008 (UTC)[reply]
I'm really grateful for the above answers - very insightful - and thanks very much for them. My reason for asking was not for my benefit - I am 61 - and have never been asked to prove my age - not even though I frequently visit the USA, including LV - when buying and consuming alcohol! But I know a young guy who works in a local Nike Store here in Scotland UK who is visiting California in a few weeks' time with his 21+ friends, and he asked me the above question. It's a shame really as he really is a mature-looking and adult-behaving 20+ Physical Fitness Student and Instructor whose last thought would be to abuse alcohol. Not forgetting that here in the UK it is legal for him to drink and purchase alcohol from age 18 - and also to gamble in licensed gambling premises, though individual shops, supermarkets, casinos etc, may legitimately refuse to serve anyone who LOOKS to be under ANY age they decree, even 24, obviously to protect their servers from having to "guesstimate". But at least now I can advise him as to the likelihood of his apparent majority being challenged. Again, many thanks.92.8.18.90 (talk) 19:05, 12 April 2008 (UTC)[reply]
Hi. You get carded to buy an M-rated game? I know people who are barely teenagers who play M-rated games. Also I know someone who apparently knows or knows someone who knows or has heard of a six-year-old who plays an M-rated game. Are the regulations stricter in some places than others? Surely a 10-year-old or 13-year old is not going to look 17. Or maybe the game was produced years ago and being rated M then doesn't mean it's M now? Lots of teenagers play halo 3 for example. Thanks. ~AH1(TCU) 01:32, 13 April 2008 (UTC)[reply]
Here we have to prove we are at least 18 if we look under 21, just in case. Maybe it's like that in America too. HS7 (talk) 10:49, 13 April 2008 (UTC)[reply]

The dimensions of a US $5 bill

In cm's or inches. Don't have one handy, and I can't find it anywhere, thanks! --Nathan12343 (talk) 22:34, 11 April 2008 (UTC)[reply]

The bills are all the same size, you can measure any denomination. The $5 bill in my wallet looks like it's about 6" by 2.5". Useight (talk) 22:55, 11 April 2008 (UTC)[reply]
I have some US money here. Most of the bills I measured are 6+1/8 inches long or just over, and 2+19/32 inches wide (that'd be 15.56 x 6.59 cm). There is some slight variation because of the effects of humidity and crumpling on the paper. I can't find official dimensions at moneyfactory.gov, where I would expect to see them somewhere, but the article United States dollar gives the dimensions as 6.14 x 2.61 inches (that'd be 15.60 x 6.63 cm). --Anonymous, in Canada, 00:44 UTC, April 12, 2008.
Wow, 6.14" by 2.61"? I was really close. Useight (talk) 00:16, 12 April 2008 (UTC)[reply]
Our article on United States dollar states " ...small-sized notes, introduced that year, measure 6.14 inches (156 mm) by 2.61 inches (66 mm) by 0.0043 inches (0.11 mm)." Consider using the search box for simple factual queries of this nature. --Cookatoo.ergo.ZooM (talk) 20:34, 12 April 2008 (UTC)[reply]
Er, yes, that's what I said it says. Thanks for confirming. :-) --Anonymous, 22:10 UTC, April 13, 2008.

April 12

Alternative medicine

From the medicine page: Medicine is the science and "art" of maintaining and restoring human health through the study, diagnosis, and treatment of patients. If this is the defintion of medicine, which contains both an explicit nod to the scientific method and an implicit one to the idea that medicine actually works, why is alternative medicine allowed to call itself medicine at all? I don't want to start a debate, but once alternative medicine has been shown to work in clinical trials it simply becomes medicine, and the active ingredients causing it to work are likely to be marketed by "Big Pharma". So why is it not illegal for pratices such as homeopathy and naturopathy to advertise themselves as "medicine" in any form? surely that is at the least false advertising? Michael Clarke, Esq. (talk) 01:18, 12 April 2008 (UTC)[reply]

You're right, but the problem is that nobody really enforces that because it isn't considered an issue, and would hardly go very far on court. Here in Brazil, homeopathy is officially recognized as a medical practice, and if you took it to court the judge would laugh at you for not believing the "wonderful" homeopathy. So, I believe that in the end the number of the people who see the problem is too small to change anything, especially something as wild and uncontrollabe as language itself. — Kieff | Talk 03:11, 12 April 2008 (UTC)[reply]
Because it's read as "alternative medicine" not as clinical mainstream "medicine"? The problem with the view (that the word medicine implies sameness) is that though something can be proven in clinical trials, it doesn't necessarily follow that it is taken up and used in the field of clinical medicine as a treatment. It only affirms its validity and may be safely incorporated in a clinic or practice, eg a medical doctor who also offers acupuncture, herbal solutions etc. These alternatives are tested and gaining acceptance, but are not meant to replace scientific medicine. As you say, "Big pharma" puts big bucks into anything that passes safety/effectiveness trials because it's always onto the next saleable thing and doesn't necessarily mean it's medicine as we know it. Julia Rossi (talk) 03:28, 12 April 2008 (UTC)[reply]
Because it's not really in people's best interest to convince most people that alternative medicines don't work. The placebo effect is extremely powerful and useful, good 'alternative medicines' are cheaper and less harmful than taking a drug you don't need and often provide a pleasant experience that adds to the placebo effect, and it takes some of the strain off the conventional system when people with colds press a crystal to their head rather than booking an appointment with their GP and demanding antibiotics. As long as the people who need to know (actual doctors, policy makers, drug manufacturers, etc) know that these things work on the placebo effect, as long as it's relatively easy to find out that it is the placebo effect at work, as long as there is sufficient regulation that harmful treatments are prevented and nobody claims to heal actual, serious illnesses with the alternative medicines (so that people who need conventional treatment get it), there is no benefit to convincing most people of the truth. There is even a disadvantage, in that the placebo effect will be much less effective. 79.66.105.94 (talk) 00:42, 14 April 2008 (UTC)[reply]

Cashing US money orders while in UK

So, you're in the UK, but you have a US Domestic money order. How do you cash it? --Alecmconroy (talk) 02:33, 12 April 2008 (UTC)[reply]

Short answer - I'm not sure you can, unless a Bureau de Change will accept them. If they did, the exchange rate and commission would be crippling, and the Money Order would probably be pretty worthless. Your best bet would be to contact a BdC before you travel to the UK, and see if they will take them. If people want to send you money whilst you're in the UK, Wire Transfers or payments to your bank account would be better (assuming you have a bank card with a VISA / MC symbol on it, in which case you can draw cash at any UK ATM) Samilong (talk) 13:41, 12 April 2008 (UTC)[reply]
Unless the US Money order is for a large sum, the fees and exchange rates make this very uneconomic. Use a different method of sending/receiving cash. Exxolon (talk) 23:56, 13 April 2008 (UTC)[reply]

Spiritual beliefs and educational background

Is the discipline of a person's post-secondary education correlated with any aspects of their spiritual beliefs, if ethnicity is controlled for? NeonMerlin 03:25, 12 April 2008 (UTC)[reply]

Hi NeonMerlin, are you asking if beliefs depend on/are affected by culture and education? Julia Rossi (talk) 03:33, 12 April 2008 (UTC)[reply]
Or is it that their choice of discipline corresponds to their belief system (cultural background aside). JR
There are some correlations, yes. This study found, for example, that biologists in the National Academy of Scientists were more likely to be agnostics and atheists than physicists, for example. Whether there are detailed studies out there, I'm not sure, but you'd expect some correlations to be present (it's no surprise to me that biologists would be more atheistic than physicists; the former spend a lot of time looking at how very small and wet things happen without intervention, the latter spend a lot of time contemplating things on so small and so large a scale as to be sublime). And of course there are disciplines like Theology and Jewish Studies which are going to be magnets for certain beliefs. --Captain Ref Desk (talk) 16:55, 13 April 2008 (UTC)[reply]

Car radio replacement

My father drives a 1989 Nissan Sentra and recently, the radio - specifically the audio but not the clock according to the shop - no longer works. I beseech your wisdom on replacing the dead radio. --Blue387 (talk) 07:56, 12 April 2008 (UTC)[reply]

I like 'beseech' even better than please!--Artjo (talk) 08:06, 12 April 2008 (UTC)[reply]
And I thought I would never see that word anywhere else other than Shakespeare... OK back on topic: Just take your car to your local motor servicing centre auto-electrician (I think that's what they're called) and get them to replace your radio with a new one. --antilivedT | C | G 09:43, 12 April 2008 (UTC)[reply]
Go to most any electronics store (Best Buy, Circuit City, Radio Shack even!) and get a new radio, get the harness and mounting brackets and such, and put a new one in yourself. If you like, you can also go to crutchfield.com and order one online. They'll send you instructions on how to put it in too if I'm not mistaken, otherwise they have a tech service line that you can call for help. It's not that hard to put in a car stereo. Or is it really that necessary to get a stock replacement? Dismas|(talk) 13:07, 12 April 2008 (UTC)[reply]
I'm guessing that saving money is important to anyone who drives a 19 year old car. The cheapest solution is to get a portable radio that can run off the cigarette lighter (you could run off batteries, of course, but that would get expensive). You can also buy a device that plugs into the cigarette lighter and gives you a normal home electrical outlet, for use with a radio that only has a regular plug. I'd put the radio on the floor of the passenger side, to prevent it from falling and being damaged in an emergency stop. The advantages of this setup are:
1) You may already have a portable radio/CD player/MP3 player, so don't need to buy one.
2) You can remove the portable radio when parked, both to use it and to prevent theft.
3) No installation is required.
4) If you end up getting rid of the car, you can easily keep the radio and converter.
5) You can easily upgrade to new technology by just unplugging the old device and plugging the new one in.
6) The 19 year old speakers in your car may be showing their age, too, and this method bypasses them. StuRat (talk) 13:42, 12 April 2008 (UTC)[reply]
Another option, primarily for a vehicle driven most often by a single person, is to wear something like an iPod when driving. I used a predecessor, a Walkman, when I had to drive a car with just an AM radio and single speaker apparently only capable of producing a whining treble sound. StuRat (talk) 13:51, 12 April 2008 (UTC)[reply]
Safety warning on the above suggestion: you should not drive while wearing an iPod or similar device as it prevents you hearing necessary sounds such as sirens of emergency vehicles or horns of other vehicles. What you can do is get a fairly cheap device that plugs into the iPod and transmits the sound on a radio frequency, which you can then tune the radio to. Advantage - all passengers can hear it, plus safer option, etc. Disadvantage - of course the radio doesn't work anyway, which is the problem! BTW, it's not that common for the audio on radios to stop working like this, these things seem to usually keep going forever - how sure are you that it's not a problem with the speaker/s, or the wiring to the speakers, rather than the radio itself? Perhaps even the fuse on the speaker circuit has blown (I'd assume not the radio circuit if the clock still works). If you can check that out it could be the cheapest fix of all. Another suggestion, especially if you want to keep the original look of the car, is go to the wreckers and get a replacement from a wreck of this model - this would usually be pretty cheap, I'd guess under $20, assuming you can locate a suitable wreck. --jjron (talk) 15:57, 12 April 2008 (UTC)[reply]
US translation: "go to the wreckers" = "go to the junk yard". As for iPods blocking outside sounds, it doesn't do that if the volume is reasonable. And, conversely, a high powered car stereo will also block outside sounds if cranked all the way up. Then there's the added danger of all four wheels bouncing off the ground as the "music" plays. :-) StuRat (talk) 18:57, 12 April 2008 (UTC)[reply]

Midway Atoll in Hawaii area of Pacific Ocean.

Sirs> > I would appreciate if you would point me in the correct direction - I recently> saw an item on UK television regarding the 'rubbish dump' in the vicinity of> Midway Atoll. I am interested in finding out more about the problem with a> view to possibly doing some volunteer work to help improve the situation.> > I would appreciate if you would advise who I should contact. —Preceding unsigned comment added by 81.105.179.16 (talk) 08:40, 12 April 2008 (UTC)[reply]

Our thread[19] above under Bigger than Texas has a link to the Blue Planet Society[20] as an example of groups who might have projects, a question like yours is deflected here[21] and there's a blog of someone who got involved here[22] that mentions Matt Brown, deputy manager of the Midway Atoll Wildlife Refuge which sounds central. Julia Rossi (talk) 12:26, 12 April 2008 (UTC)[reply]

Bulgarians

Is it true that Bulgarians:

  1. nod their heads when they mean no?
  2. show soft porn at bus stops on giant screens?
  3. believe that local Boza beer increases the size of women's breasts?
  4. buy tigers to ward off thieves?
—Preceding unsigned comment added by Chocolate dessert (talkcontribs) 14:40, 12 April 2008 (UTC)[reply]
  1. It could be true yet it could be false, you'll never know. Ericthebrainiac (talk) 15:00, 12 April 2008 (UTC)[reply]
  2. Lord, no. Because that is an international crime. Ericthebrainiac (talk) 15:00, 12 April 2008 (UTC)[reply]
  3. Boza beer doesn't make breasts bigger, but breast surgery does. Ericthebrainiac (talk) 15:00, 12 April 2008 (UTC)[reply]
  4. Tigers could ward off thieves, but keep in mind that they will and the can turn on you on a dime like Kip turned on B.G.. Ericthebrainiac (talk) 15:00, 12 April 2008 (UTC)[reply]
  1. <shake head> Clarityfiend (talk) 15:54, 12 April 2008 (UTC)[reply]
I've heard the first one before, but I've no idea if it's actually true. I kind of hope the others are. Especially 4. Michael Clarke, Esq. (talk) 16:36, 12 April 2008 (UTC)[reply]
It would appear that 2 is also true: [23] [24] [25] D0762 (talk) 17:39, 12 April 2008 (UTC)[reply]
Wouldn't 4 be rather expensive overkill? Clarityfiend (talk) 17:42, 12 April 2008 (UTC)[reply]
Number 1 seems to be true as well, from wikitravel: "It is also important to remember the fact that many Bulgarians - contrary to most nationalities - shake their head for Yes and nod for No!" [26] And, from our very own wikipedia article Boza#Controversy "Boza allegedly has the ability to enlarge women's breasts" [27]. And, as amazing as it may seem, 4 also appears to be true: "With a series of burglaries rocking the village of Selisten Dol in western Bulgaria, residents found a novel way out. They purchased a tiger cub from the Sofia Zoo to guard their village. The ploy worked. The thieves have stopped thieving around in the village." [28] D0762 (talk) 17:49, 12 April 2008 (UTC)[reply]
No idea if this book is reliable or not, but it claims that Bulgarians do indeed buy tiger cubs to ward off thieves, specifically that the village of Selisten Dol bought a tiger cub as a 'guard cat' Michael Clarke, Esq. (talk) 17:49, 12 April 2008 (UTC)[reply]
Oh and the page for Boza states that Boza is indeed claimed to increase breast size, which is referenced by this story at news.au Michael Clarke, Esq. (talk) 17:54, 12 April 2008 (UTC)[reply]
Ha, sorry D0762, only just noticed that we both provided the same links! Shows the power of google, I guess. Michael Clarke, Esq. (talk) 17:57, 12 April 2008 (UTC)[reply]
OR, but adequate quan-titties of any alcoholic beverages seem to result in breast augmentation and miraculous cosmetic ad-hic surgery in any lustfully observed objects. --Cookatoo.ergo.ZooM (talk) 19:09, 12 April 2008 (UTC)[reply]
I remember hearing that the Greeks also shake their heads for yes and nod for no. But that was on some TV documentary many years ago and maybe it was talking about Bulgarians. Adam Bishop (talk) 02:40, 13 April 2008 (UTC)[reply]
I've only ever heard this of Indians (from India), but one never knows. -- JackofOz (talk) 07:33, 13 April 2008 (UTC)[reply]
Expect that the dedicated Boza beer drinker might start to see bigger breasts everywhere. Julia Rossi (talk) 08:04, 13 April 2008 (UTC)[reply]

Was Gomer Pyle just effeminate or was he also a homosexual? 71.100.160.37 (talk) 16:51, 12 April 2008 (UTC) [reply]

He was an innocent. You hardly ever see them anymore in the industrialized world. His self-image (a term he would not have understood) was untainted by programmed preconceptions of masculinity. We hard-boiled realist philosophers too easily mistake gentleness and piety for effeminacy. He adored women and saw them not as lust objects but as a complete element of his world, those lofty things that were to be respected, cherished, protected, courted, and married. There are worse ways to be. --Milkbreath (talk) 19:05, 12 April 2008 (UTC)[reply]
I'd rather be Gomer than Sargeant Carter. StuRat (talk) 20:33, 12 April 2008 (UTC)[reply]

What is the difference...

...Between melodic death metal and regular, (unmelodic?) death metal? I'm just curious. I don't really listen to death metal aside from a tiny bit of Opeth (The Drapery Falls is one of my favorite songs) and a few obscure Christian death metal bands. Can I have some examples showing the difference? Like lyrical comparison or something? Just tell me. MalwareSmarts (talk) 17:14, 12 April 2008 (UTC)[reply]

There are Christian death metal bands!? 81.187.153.189 (talk) 18:39, 12 April 2008 (UTC)[reply]
Indeed there are. See Christian metal#Subgenres. Mad but true. And to the OP, I would hazard a guess that the difference is the use of melody in melodic death metal, although I think the extensive number of genres in both metal and electronic music are often innapropriate and restrictive. Who needs Techstep New Wave Psy-Jungle anyway? And who will listen to Extreme Viking Glam Metal but not Avant-garde Sludge Metalcore? Those pages do make for some hilarious reading, though. Michael Clarke, Esq. (talk) 19:04, 12 April 2008 (UTC)[reply]
Squire Clarke's answer is pretty straight forward. It is true that the subdivisions of musical genre are somewhat arbitrary. However, bands classified as melodic death-metal are more likely to have a melodic structure outside of guitar solos, and a more melodic vocal line (rather than full scream). Non-melodic or thrash death-metal is more based on the rhythmic variation of the guitar riffs and drumming patterns, and have a more percussionist vocal style (i.e. based on rhythm rather than pitch variation). Steewi (talk) 02:57, 14 April 2008 (UTC)[reply]

Not seeing the whole picture

Are there any documented instances of news footage being misunderstood because objects that were caught on camera were cut off by the overscan on viewers' TV sets? NeonMerlin 17:15, 12 April 2008 (UTC)[reply]

Flat Rock Indian site in Barnesville, OH 43713 area

Hello. I am looking for information about Flat Rock in or around Barnesville, Ohio 43713 where there is supposed to be a great deal of native American Indian history and evidence. I hear it is west past the hospital, but was unable to find it on my last journey. Please reply yo jmichaelfader at yahoo dot com —Preceding unsigned comment added by 67.163.241.60 (talk) 17:41, 12 April 2008 (UTC)[reply]

Do you mean Flat Rock, Michigan ? StuRat (talk) 19:52, 12 April 2008 (UTC)[reply]

history of old German small arms manufacturer D.W.M.

Can anyone give an overview ? Dates etc ?--Gerrycan (talk) 19:31, 12 April 2008 (UTC)[reply]

There's a brief overview in WP's article on Deutsche Waffen- und Munitionsfabriken with links to articles on some of the arms and people involved. ---Sluzzelin talk 20:17, 12 April 2008 (UTC)[reply]
DWM (Deutsche Waffen- und Munitionsfabriken AG) existed from 1896 to 1945 in Karlsruhe. The plant was destroyed by the British at the end of WWII. There is an article at http://www.dwm-schlutup.de/ in German which contains numerous pictures. This article speaks of more than 10,000 staff, mainly forced labour, who produced ammunition, hand grenades and canons. The entire plant of some 190,000m2 was camouflaged by the roofs being planted with grass and shrubs.
--Cookatoo.ergo.ZooM (talk) 21:33, 12 April 2008 (UTC)[reply]

cars

Why are mechanics jerks? is it the rough nature of maunal labor that furments their attitude? bskinner —Preceding unsigned comment added by 68.167.233.154 (talk) 19:40, 12 April 2008 (UTC)[reply]

I believe most mechanics are jerks because most mechanics dont make enough money, and are always dirty with car grease and feel like they get asked stupid questions all day long, and most mechanics i have encountered are "rednecks" But dont take that as a real answer just my personal opinion --Nick910 (talk) 20:04, 12 April 2008 (UTC)[reply]
A lot of it may be class differences. Mechanics, being blue collar workers, often have contempt for white collar workers, as they represent the banker who repossesed their home, the lawyer who sues them for child-support, and the doctor who tells them their drinking, smoking, drugs, and unprotected sex may be the cause of their health problems. Many blue collar workers, like factory employees, rarely come in contact with white collar workers on the job, so it's not an issue there. On the other hand, auto mechanics often come in contact with white collar workers, since blue collar workers will often fix their own cars. If you put on jeans, rub some dirt under your fingernails, mess up your hair a bit, and research the problem on the car first so you can "speak their language", they may treat you better. I intentionally skip shaving on days I need to see a mechanic.
Another factor may be that people with people skills gravitate to other professions, like sales, where those skills can earn them money. The management at many auto repair places also intentionally engages in deceptive practices, like promising they can "replace your muffler for under $20", only to tell you it will cost hundreds to replace the entire exhaust system, which, of course, it always needs once they get it up on the lift.
Some auto repair shops have a person specially designated to talk with the customers, who wears a tie, can speak proper English, and is polite. The owner presumably realizes that they will lose customers if rude mechanics answer customer questions by spitting on the floor, blowing a fart, laughing at the question, and leaving. My personal favorite story is when I was in a junk yard looking for a seat belt retractor mechanism. I was inside the car checking the mechanism to see if it was what I needed. I heard splashing water outside and then noticed the mechanic was pissing on the car tire. I suppose I should be grateful he didn't have a large lunch or I would presumably have gotten an extra special treat that day. StuRat (talk) 20:08, 12 April 2008 (UTC)[reply]
I suspect it is class issues as well as power issues. Most mechanics can probably tell right away who knows something about a car and who doesn't. Someone who doesn't know about a car is in a distinct disadvantage when talking with a mechanic—a car is very complicated, and repair can be very expensive, so most people going to a mechanic are somewhat afraid of things going wrong and are going to put up with behavior that they'd otherwise find objectionable. That being said, I've met friendly, helpful mechanics—I don't think its inherent to the profession, though it might be common. A good, trustworthy mechanic is hard to find, but worth it even if they cost a bit more. --Captain Ref Desk (talk) 20:31, 12 April 2008 (UTC)[reply]
Yes, talking with an "expert" is frequently problematic. For example, talking with a doctor you often get the distinct impression they don't care what you say at all, and consider you an ignorant waste of their time. I also get that attitude from some "experts" at Wikipedia, unfortunately. StuRat (talk) 20:40, 12 April 2008 (UTC)[reply]
My garage and mechanics are polite, friendly and quite happy to explain any work they are doing on my car, give me a full breakdown of parts and labour and won't do any work that isn't required or I haven't agreed to. Maybe I'm just lucky :) Exxolon (talk) 23:54, 13 April 2008 (UTC)[reply]
To increase the luck, it's often helpful to slip in a question related to "best practices" but in their language, such as I'd like to see the part causing all that trouble, is that possible? etc... Sometimes a mechanic like mine gets frustrated to be needed so heavily by the non-mechanic part of the population. Contempt there seems to rise in proportion to expensive car+rich client+reluctance to pay. Mine's an early T.corolla that's (to him) no sweat. Julia Rossi (talk) 00:38, 14 April 2008 (UTC)[reply]

airline blues

why--68.167.233.154 (talk) 19:51, 12 April 2008 (UTC)are all the airlines suddenly going out of busniess ?[reply]

I'd say the main factor is fuel prices. As airlines raise fare prices to match, fewer people fly, and then fewer airlines can be supported by the lower number of passengers. Those which were borderline when fuel was cheap are the first to go under. Those that were making profits then may survive. StuRat (talk) 19:56, 12 April 2008 (UTC)[reply]
Hi. Uh oh. So what will happen if oil runs out?... ~AH1(TCU) 23:44, 12 April 2008 (UTC)[reply]
Well, airplanes would need to move to alternative fuels. The US air force is already planning on using fuel made from coal, for example. Unfortunately, changing over an airline fleet to use a new fuel is also quite expensive, so that would force the weaker airlines out of the market, too. StuRat (talk) 04:22, 13 April 2008 (UTC)[reply]

miscellaneous fact about youth automobile ownership in the United States

Can I--or even better you, as my computer search skills are rudimentary (I'm old)--determine the percentage of youth under the age of twenty-one who own their own cars in the United States? paulzipPaulzip (talk) 20:29, 12 April 2008 (UTC)[reply]

Here's a start[29] if you'd like to do the searching/reading/eye ruination. Cheers, Julia Rossi (talk) 08:27, 13 April 2008 (UTC)[reply]

Bird Poop

<moved to science desk Julia Rossi (talk) 22:12, 12 April 2008 (UTC)>[reply]

Its All About Breakfast

I read in an article that those who do not eat breakfast will become obese in their lifetime. Is this true? And if this is true does that mean that if you're someone like me who doesn't eat breakfast that I will to become obese or overweight? I don't eat breakfast because I don't or forget, but because breakfast makes me sick to my stomach. I have to at least wait three hours before I eat breakfast. Three hours prior to when I wake up.

Thank You

Always

Cardinal Raven

Cardinal Raven (talk) 21:29, 12 April 2008 (UTC)Cardinal Raven[reply]


I sympathize with you Cardinal, if i eat breakfast right when i wake up i get bad indigestion/stomach, so i don't eat breakfast much, but i am now 22 and i have not been eating breakfast for a long time since i was about 9 or 10 according to what i asked my mom, and right now i am 6foot 4 and weigh 200 pounds. Of course none of this is medical research, but this is my personal experiance --Nick910 (talk) 21:35, 12 April 2008 (UTC)[reply]

I'm in the age group of 18-29 and I weigh in this group 100-110. I just don't want my body to go froop because I can't eat breakfast or end up eating more because of not eating breakfast. But then again I also have to trust my body. It will tell me when its full and when its hungry. Though its kinda scary cause I've read a lot of articles about how someone could trust their body for so long until it goes against them. I don't want it to go against me.

Always

Cardinal Raven

Cardinal Raven (talk) 21:44, 12 April 2008 (UTC)Cardinal Raven[reply]

I also sympathize with you dear Cardinal. I have never really eaten breakfast because of the same reason Nick910 states. I am 18 I'm 5'4 and weigh 106 lbs and I have not gained/lost a single pound from not eating breakfast nor have I became obese... SlaveofBetrayal (talk) 21:47, 12 April 2008 (UTC)[reply]
I have no sources to back up my claim but the writer of whatever you read may feel that by not eating breakfast, you may be setting yourself up for either A) gorging yourself at lunch and eating more than you should or B) finding unhealthy snack food at work/school when you finally do eat. Dismas|(talk) 21:52, 12 April 2008 (UTC)[reply]

B is most like me. I find the most unhealthy food to eat. The problem though is that if I haven't had breakfast and I haven't anything to eat. My stomach needs something inside of it and sadly the only thing is something unhealthy. Also its very hard to find lunch (brunch) when I'm really hungry cause nothing sounds good or nothing sounds appetizing enough. I understand where you guys come from. For those who can't eat breakfast.

Always

Cardinal Raven

Cardinal Raven (talk) 21:59, 12 April 2008 (UTC)Cardinal Raven[reply]

Hi. Well, I heard that not eating breakfast will cause your body to go in survival mode (unless you're used to missing breakfast, I guess), and when you have lunch, the body says, "oh, great, I finally get food, so I'd better fatten up in case I need to go without food for another morning", but I don't have a source for this. Also, there are blends and drinks avalible for those too hurried for breakfast, and some breakfasts can be made so that they only take 5 - 10 minutes. Hope this helps. Thanks. ~AH1(TCU) 23:43, 12 April 2008 (UTC)[reply]
Not sure but are you talking about Starvation mode?--Lenticel (talk) 02:57, 13 April 2008 (UTC)[reply]
I have always eaten breakfast (in fact even if I wake up at 2 in the afternoon I still need to eat "breakfast" food, which amuses other people greatly), and I am, well maybe not obese, but certainly not skinny... Adam Bishop (talk) 02:38, 13 April 2008 (UTC)[reply]

Who else's lifetime would you become obese in? Daniel (‽) 18:13, 13 April 2008 (UTC)[reply]

I can't eat first thing either.I only put weight on when I became immobile with health problems.hotclaws 06:56, 14 April 2008 (UTC)[reply]

technocracy

Is it possible politicians in the future will be replaced by computers? Obviously a very speculative question and i'm not saying all politicians, but say for example here in NZ we have 120 mps - wouldn't it be viable to have one or so replaced by a computer that is fed variables and spits out a response free of bias? They would unaligned, neutral, maybe callous - but fair? What would be reasons for non-implementation? I guess the programmer would have to be trusted but i'd vote for them - as long as their gui was nice. Boomshanka (talk) 22:36, 12 April 2008 (UTC)[reply]

A scheme (not publicised widely) to replace all public personae with computer simulated bots has been implemented and was completed in 1984. Only a few backward countries still use humans in politics, films or sporting events.
As the last update of the hardware of H.S. was deployed some 150,000 years ago, the model is considered to be obsolete and is neither supported nor serviced. There is a limited market amongst nostalgic collectors of pre-singularity organic artifacts. --Cookatoo.ergo.ZooM (talk) 23:14, 12 April 2008 (UTC)[reply]
Computers are only as objective as the data that is put into them. The idea that mechanical decision-making would produce fair or even prudent ruling, much less "objective" ruling (and what, exactly, does that word mean in politics?) is naive. Almost no questions in politics are purely factual in a simple yes/no fashion. Should we do more to save the environment or to bolster the economy? There's no value-free answer to the question. Indeed, "values" themselves—not "objectivity"—are often what we try to hope for in a politician. --Captain Ref Desk (talk) 00:36, 13 April 2008 (UTC)[reply]
Well, in way, perhaps. The goal of representative democracy, in theory, is to get the representatives to vote as their constituents would vote. A computer could certainly do that, based on polling what the constituents think about various issues. In practice, this would essentially be direct democracy, where the people vote directly (quite possibly via computer) on each issue. This would eliminate much of the corruption associated with reps who are really looking out for their own interests at the expense of their constituents and/or the country as a whole. StuRat (talk) 04:15, 13 April 2008 (UTC)[reply]
The main job of representatives in parliament isn't to vote. They're there do debate, discuss, propose, negotiate and elaborate over budgets, laws etc. If all they did was vote, they could easily be replaced by any number of systems that polled/sampled/represented the will of the people. Somebody would have to decide on what they were to vote on, though, and that is where representing the will of the people get far more complicated. Imagine an automated, statistics based parliament doing the voting on proposals by a government of humans. That government would quickly learn how to play the parameters to obtain 50%+ for anything they propose by ammending smaller issues that brings in a few extra percent etc. With a fully predictable parliament, a government can basically do what they want. /85.194.44.18 (talk) 09:55, 13 April 2008 (UTC)[reply]
Indeed. Politicians and civil servants have to spend a lot of time becoming familiar with a lot of things the general population have neither the inclination nor the time to become fully familiar with. How they eventually tweak proposals and laws, and how they vote, is dependent on this. When you vote someone in, you hope they'll do what you would do if you knew what they knew, not that they'll vote the exact way you'd vote if someone put you on the spot. When parliament debate how much to spend in different areas of the NHS and how autonomous trusts should be, I want someone who knows much more than me on the topic to spend time finding out about it and considering the ramifications of various decisions, then make decisions based on similar values to me. I don't want a machine to vote yes or no on some simplified situation based on what it thinks I want, nor do I want decisions made by polling the snap decisions of people like me who don't actually know a lot about the topic. 79.66.105.94 (talk) 00:25, 14 April 2008 (UTC)[reply]
It would be nice to think that representatives take the time to read every detail about every issue and become experts on everything, but it's simply not the case. For example, I seem to recall that the US Congress was given the opportunity to read the full report on Iraq prepared by the CIA before voting to give Bush the authority to declare war, but hardly any actually bothered. And, being mostly lawyers, reps tend to do quite poorly when asked to judge the relative merits of scientific endeavors. If everyone could vote (either directly or via some computer system), I would expect to see scientists mostly vote on matters of science, educators on matters of education, etc. Hopefully then we would actually get some real expertise in the decision-making process. StuRat (talk) 17:00, 14 April 2008 (UTC)[reply]

Home improvement

I was wondering if I could do home improvement to my terrible looking apartment without pissing off the complex owner. I would like to install better cabinets so that I can hold more stuff. I would also like to change the door handles on my doors. Is it possible to do all of this without making the owner mad? What can I do to make him agree to letting me do these things to my apartment room?--SlaveofBetrayal (talk) 23:45, 12 April 2008 (UTC)[reply]

Try asking him, he might be more cooperative than you think. He'd benefit from those things too, as long as you don't have awful taste. Michael Clarke, Esq. (talk) 23:52, 12 April 2008 (UTC)[reply]

as long as you don't have awful taste This is a key importance. But yes just ask the man he might be willing. Maybe he thinks its just as ugly. 71.142.208.226 (talk) 00:07, 13 April 2008 (UTC)Cardinal Raven[reply]

The last place I lived, our landlord told us he'd compensate us for materials if we repainted. He'd have had to approve the color choices first but that was all. We never did paint the place though. Weren't there long enough to justify. Dismas|(talk) 01:57, 13 April 2008 (UTC)[reply]

April 13

Legalization

I heard that there is a high chance that marijuana will be legalized within the next twenty years (in the US). How much truth is there to this? 69.40.248.234 (talk) 00:31, 13 April 2008 (UTC)[reply]

I myself wouldn't know, but I highly doubt it. To legalize marijuana would be the worst thing on the planet. The worst plan on the planet.

Cardinal Raven (talk) 00:48, 13 April 2008 (UTC)Cardinal Raven[reply]

Actually it would be a great idea, there is a high possibility that with in the next 10 15 years that it will be decriminalized, it already is in some states, and most countries in Europe it is perfectly legal, keep a look out, until then, keep you cell phone ready. --Nick910 (talk) 01:33, 13 April 2008 (UTC)[reply]
In most countries in Europe it is perfectly legal? I think you're mistaken. It's legal in some but definitely not a majority, decriminalised in others although even though probably still not a majority. Bear in mind that Europe has a lot of countries. The enforcement of cannabis offences in Europe even where it remains a criminal offence is probably generally a lot weaker (if enforced at all) then the US, but that doesn't mean it's decriminalised, let alone legal. Nil Einne (talk) 13:43, 13 April 2008 (UTC)[reply]
Trying to predict the future is a pretty futile endeavour. You may however find our articles
contain a number of useful references for further information on the topic. Note also the distinction between legalization and decriminalization in discussions on the topic.
Incidentally, I strongly urge any other editors who might wish to add to this thread to remember that this is the Reference Desk, and that we should aim to provide readers with links to useful, relevant sources. Editorializing or extended debate are neither helpful nor necessary. TenOfAllTrades(talk) 03:12, 13 April 2008 (UTC)[reply]
I think this may well happen. The biggest reason is simply demographics. Many people who grew up in the drug culture of the 1960s are now serving in government offices. Convincing people who have personal experience with marijuana that it's harmful and should continue to be banned is much harder than convincing people who don't know anything about it. Even former President Clinton admitted to using marijuana. StuRat (talk) 04:06, 13 April 2008 (UTC)[reply]
The main problem with making marijuana legalized is there not anyway for it to be used without the user becoming impaired. Meaning, you can drink an alcoholic beverage without becoming impaired. You cannot smoke marijuana without becoming impaired. —Preceding unsigned comment added by 76.225.133.60 (talk) 07:32, 13 April 2008 (UTC)[reply]


I'm not sure what your definition of impaired is, but alcohol must be the most 'impairing' drug available. Death and injury due to these legal drugs, alcohol and tobacco, outweighs by orders of magnitude the damage wrought by drugs like cocaine and heroin, and cannabis has been shown to be fairly benign in all scientific testing over the last half-century. Its criminalisation is more a political matter than a medical one. 121.44.244.114 (talk) 07:49, 13 April 2008 (UTC)[reply]

You're reading too much into the statement. So, I'll explain. You can go out to eat, stay at home, go to a party or sporting event and drink alcohol, without becoming imapired. The way you do that is to avoid drinking too much alcohol.
You can go out to eat, stay at home, go to a party or sporting event and smoke marihuana, where the only intent is to become impaired. After smoking marijuana, if you did not become impaired (which means to become euphoric or high), then you would not want to smoke any more and would probably be angry at whoever sold it to you. What would be the point?
Therefore, if marijuana was used in a social setting, like the ones mentioned above, then you would need to sit around for a few hours until you were no longer impaired, in order to leave and not drive while impaired.
This would be the definition of Impaired: Diminished, damaged, or weakened: Functioning poorly or incompetently: Having a physical or mental disability.
I'm not saying either one should or should not be legal or illegal. I'm just adding a fact that most people seem to forget. —Preceding unsigned comment added by 76.225.133.60 (talk) 08:10, 13 April 2008 (UTC)[reply]
I don't buy it--frankly I think you're flat wrong. If you're drinking enough alcohol to get the positive effects (reduced anxiety, general sense of well being, reduced social inhibitions, etc) then you're also drinking enough to impair you in measurable ways (reaction time, for example). Maybe not enough to keep you from driving, but only because we allow people to drive at a certain level of impairment (as indeed we must for practical purposes, for if a driver had to be in top shape to legally drive, how many of us could get to work on a typical Monday?). --Trovatore (talk) 08:34, 13 April 2008 (UTC)[reply]
Another comment (sorry I can't find a wikipedia article on this topic) -- impairment depends on the task at hand. It is well known that cannabis can, not always but often, enhance musical perception. It may be true that if your task at hand is to drive a car, cannabis may result in impairment. But if your task is to compose music and play an instrument with other musicians -- well there are many musicians who have vouched for the power of cannabis in this regard. I searched wikipedia for an article on cannabis and music, but failed to find one. Isn't there one? The number of musicians who have vouched for the power of cannabis is at least notable. 09:30, 13 April 2008 (UTC)

As some article I recently read put it -- just a decade or so ago a US presidential candidate had to go through great pains to claim that although they smoked cannabis they "did not inhale". Today, in the US, presidential candidates need no such weaseling and simple say they did in fact smoke cannabis when young -- as well as other drugs such as cocaine. And it does nothing to reduce their chances to win the presidency. Even McCain, as this article put it, expressed a kind of remorse in having to admit that he had not smoked, with the excuse that he was in a Vietnamese prison camp during the time when he would have had the greatest opportunity. If a presidential candidate can admit to committing what is still considered a serious crime without any meaningful stain on their character, one has to wonder whether this "crime" is as serious as some make it out to be. People who have committed the same crime rot in prison or run for president. Does this not sound like a farce? As for "within 20 years"? Who knows. We in the US should be so lucky, as taxpayers if nothing else. (I hope this comment is useful and not merely editorializing) Pfly (talk) 09:21, 13 April 2008 (UTC)[reply]

I remember when I was younger, way younger, smoking pot just made me sleepy. I have a terrible time sleeping now. If it was legal, I wouldn't have to take so much medication. I could take a few tokes, eat a bag of chips, and ZZZZZzzzzzz for me. —Preceding unsigned comment added by JelloTube (talkcontribs) 12:32, 13 April 2008 (UTC)[reply]
I don't know about twenty years, but I really believe that most Western democratic nations will at least be on the road to total legalisation within 30-50 years. From an economic standpoint, the war on drugs is unfeasible, and demand has increased rather than decreased in the last 10 years or so. Drugs are becoming more widely available and cheaper than ever, and it seems to me that governments will eventually realise there are major benefits to legalisation. These include the ability to tax sales on drugs (in the UK, over 60% of the price of cigarettes is tax), the fact that it would be taking the responsibilty for the supply of drugs out of the hands of criminals and therefore likely reducing drug-related crime massively, and the safety of users being ensured as there is little chance they will be buying drugs that have been cut. Of course there are downsides, including public opposition and the inability to know whether legalisation will result in more users, but if societies continue the modern trend of a slow march towards libertarianism-influenced social democracies, then it seems almost inevitable that drugs will become legalised, and almost certain that marijuana will be the first.
And as a reply to some statements above, "impaired" does not mean "to become euphoric or high". Alcohol and cannabis both have drastically different effects on the body, but both certainly qualify as impairment. I wouldn't drive a car stoned, but I certainly wouldn't do it after a few drinks (it's a moot point, I can't drive). For me the maor problem with the currently illegal drugs is that by virtue of their being illegal, they are controlled by criminals, and if you're willing to risk prison time to sell cocaine, it stands to reason that you may be more willing to use violence to enforce your business realtionships. Michael Clarke, Esq. (talk) 22:37, 13 April 2008 (UTC)[reply]

bike lock conundrum

This has me extremely puzzled, so I'm putting it out there for others to think about. I usually lock my bike to a street sign outside my house. I use a good D-lock and a cable lock, both connecting the slim pole with the bike frame. It was locked normally last night, but when I went outside today, I noticed the D-lock laying on the ground, still around the sign pole but no longer connected to the bike. The bike was also laying on the ground when usually it is stood up properly. The cable lock was still connected as normal. Upon inspection, I thought that probably the sign had been replaced due to fresh bitumen being present at the base of the sign. So I guess they saw my bike there, drilled into the base of the pole, then lifted the pole up through the locks, then kindly threaded the new pole through my cable lock when putting it into the ground, so as not to leave my bike unlocked. Then I realised, the D-lock was not connected to the bike anymore! What the hell happened? The bike is not damaged, obviously. The D-lock cannot be locked or unlocked without the key, so even if I had left it not completely locked, it could not be locked anyway. What's going on? The best I can come up with is that someone else with a similar lock tried out their key for fun, unlocked and relocked it around the pole, which is a pretty weak guess. Anyone have any ideas? 121.44.244.114 (talk) 02:38, 13 April 2008 (UTC)[reply]

I have two theories. 1 ) You were in a hurry and did not lock your bike as well as you thought. I've done this myself before. 2) You've got one of these locks that can easily be forced open with a ballpoint pen and someone is showing off. 3) Someone who's good at picking locks is showing off. These are only theories though. Maybe someone else can come up with a better theory. APL (talk) 03:43, 13 April 2008 (UTC)[reply]

Yes, I thought about these possibilities. The lock is pretty much impossible to almost lock, as you turn a key to do so. The key won't turn if it isn't locked. I also check it every time. Also, if someone picked it with a pen, etc, how did they relock it? Or would that be possible using the same trick? 121.44.244.114 (talk) 03:51, 13 April 2008 (UTC)[reply]

living off campus

hey, im looking for apartments for next school year, and im wondering if anybody who has experience with this can help me out. The ones that I think I want to live at are about a mile off campus, which seems like a long ways to me. Has anyone ever lived in an apartment that far from campus? Does it have any real negative effects or anything on your life? thanks. —Preceding unsigned comment added by 128.101.53.181 (talk) 04:03, 13 April 2008 (UTC)[reply]

I've lived a couple miles from (university) campus before. I was mostly bicycling at the time, not driving (on a campus where bicycling is a BIG thing; as in it would almost be pointless anyway to drive to campus, park, and walk to class). It can be more of a pain to get to class, and it's not as convenient to go home between classes for lunch or whatever, but you get used to adjusting your schedule to give yourself plenty of time. It's really going to be up to you, given your situation and preferences. Living that kind of distance from campus is hardly the end of the world, even if you are only walking. Keep it in mind when you create your class schedule. --Prestidigitator (talk) 04:49, 13 April 2008 (UTC)[reply]
What's the weather like where you're going to school? If it's cold or rainy, it can be a pain. Especially if you drive that mile in the rain and then have to find a place to park. Dismas|(talk) 06:26, 13 April 2008 (UTC)[reply]
"Drive that mile"??!! Richard Avery (talk) 07:27, 13 April 2008 (UTC)[reply]
Only a mile? That 15–20 minutes walk each way provides you with essential exercise.--Shantavira|feed me 07:39, 13 April 2008 (UTC)[reply]
Yes, drive. Never did the OP say that they were a fan of walking or biking, everyone seems to just assume that they would walk/bike. They may not want to walk/bike in the rain/snow. Also, we don't know what's between the residence and the university. There may be some impediment to easily walking or biking the distance. Basically, we don't know all of their circumstances. Dismas|(talk) 09:19, 13 April 2008 (UTC)[reply]
It can be an encouragement to study in between lectures in the library rather than just go home and doss about. Also, do you have a subsidised canteen? The University I went to did, and you could get a sandwich at lunchtime for little more than the cost of making it yourself, and hot meals at reasonable prices too. I used to generally go in for the first lecture and stay in until the last (sometimes studying afterwards or visiting the bar. -- Q Chris (talk) 08:34, 13 April 2008 (UTC)[reply]
In New York City it is not at all uncommon for people to have to walk a mile or so to get to a subway station. It is common for a great number of people of all ages and in a wide range of health. Now that I no longer live there I rather miss the requirement of walking a mile or so a day, or twice a day. It can be a pain when it is cold, raining, snowing, etc -- but it is, as they say, Good For You. Rather than negative effects of having to walk such distances 2 or 3, or more times a day, I rather feel it is an overall positive thing, even if most people would not do it if they had a choice. You get used to it. Then you come to appreciate it. Further, not only does it provide a minimal form of exercise, it is an opportunity to learn patience and meditative perseverance. You should be so lucky to have it foist upon you! Pfly (talk) 09:42, 13 April 2008 (UTC)[reply]
As an undergraduate, I lived about a mile from campus in snowy, rainy New England. I lived with housemates, and some other undergraduates lived similar distances from campus, so we had visitors fairly often and as much of a social life as we wanted. As others suggest, walking or bicycling that distance can really boost your mental and physical health. (Please don't drive it if you can avoid it. You and the environment will be better off if you don't.) By the way, my commute to work (still in snowy, rainy New England) involves nearly a mile of walking each way (in addition to several miles of subway/underground). Even in the rain and snow, I welcome the fresh air and exercise. Marco polo (talk) 20:29, 13 April 2008 (UTC)[reply]
If you live on-campus now, you'll want to look at the parking situation - my school has seperate permits for residents and commuters. We can only park on the streets nearby (2 hour max) or at the park'n'ride (WAY off campus) until after 2pm, so we're always rushing between classes to move the car. We're going to move closer to campus so I can bike to class instead. Kuronue | Talk 14:19, 14 April 2008 (UTC)[reply]

Log Cabin Mortar

What is the mortar used in between the logs of an old log home made of? —Preceding unsigned comment added by 64.119.61.7 (talk) 14:00, 13 April 2008 (UTC)[reply]

The stuff is called chinking and is usually a type of dried moss, Pleurozium schreberi or whatever grows in the location. --Cookatoo.ergo.ZooM (talk) 15:11, 13 April 2008 (UTC)[reply]
The most effective modern chinking is discussed here [30] and here [31]. There is not much moss used in North America anymore. ៛ Bielle (talk) 00:20, 14 April 2008 (UTC)[reply]

Hypnosis

Can people really be made to think they're chickens and the like by using hypnosis. It never rings true for me when I see stage hypnotists on tv. Are the audience members plants, just playing along for fun or genuinely that susceptible. Stanstaple (talk) 14:12, 13 April 2008 (UTC)[reply]

It has never rung that true for me either but my guess is that people are both 1. playing along to some degree under pressure, and 2. that people are just suggestible enough that through the pressure to perform they're actually going to, on some level, really feel that they are being compelled to do it, though if that pressure were suddenly relieved somehow (e.g. somebody bombed the building and everyone had to immediately leave) I imagine they'd "snap out of it" fairly instantaneously. But that's not a terribly scientifically informed opinion, though I think one can never really underestimate the ability of people to experience "real" phenomena of this sort when under pressure to perform. In looking over the hypnosis page I see that my view of it is pretty much what is known as the "social construction" theory of hypnosis. The hypnosis article has some interesting comments about "stage hypnosis" in general.
Note that I do think there's evidence that some forms of hypnosis are legitimate; I'm constraining my comments above to the stage-magic, "you are a chicken" variety. --Captain Ref Desk (talk) 15:59, 13 April 2008 (UTC)[reply]
What I've heard is that generally hypnosis increases the subject's suggestibility, to the point that they do just feel like "going along with it". However, it can't be used to make a person do things completely against their personality, and anyone trying that will tend to see them either resist, or snap out of it entirely. I think I saw it described as being able to make someone do something they didn't want to do, compared to making them do something they wanted not to do. Confusing Manifestation(Say hi!) 05:06, 14 April 2008 (UTC)[reply]
As per CRD point 1, maybe most of them are chickens pretending to be humans until the covers are off! ; ) Julia Rossi (talk) 11:28, 14 April 2008 (UTC)[reply]

Metal hand

What is the device on this girls hand called and where can I get one? [32] —Preceding unsigned comment added by 79.76.165.47 (talk) 15:15, 13 April 2008 (UTC)[reply]

I'm not an arms expert, but to me it looks like Wolverine's retractable Adamantium claw. Googling gives you some commercial and how-to-make-your-own links. ---Sluzzelin talk 16:06, 13 April 2008 (UTC)[reply]
Concur it's a Wolverine claw, Kari made it for the episode of Mythbusters where they were trying to see how many balloons it'd take to lift a 3 year old off of the ground. She used the claw to pop the balloons at the end. Nanonic (talk) 16:20, 13 April 2008 (UTC)[reply]

Building an instrument

I plan to build an instrument this summer, and I'm just wondering about the strings I should use. I'm looking for an odd, Asian sound. I was planning on using banjo strings, but is there some other strings I could use? They have to be easy to find and cheap to buy because I play my instruments hard (It's not my fault! I just started by playing bass!), and I tend to break strings. Should I use banjo strings or is there something better I could use? Thanks! MalwareSmarts (talk) 15:56, 13 April 2008 (UTC)[reply]

What kind of instrument are you building? I would say that the shape, size, material and construction of the instrument itself would have much more effect on the sound of the instrument than whether you use banjo, guitar or shamisen strings. jeffjon (talk) 18:19, 14 April 2008 (UTC)[reply]
The instrument I plan to build has no name, it's original. But it would probably be made of pine and oak because I can get that wood around where I live. I haven't really planned this much, but it should be basically a strip of wood somewhere between 24 to 30 inches glued to a box with holes in it with a string or two attached. It's not exactly a professional effort. More of a plucky toy than anything else. MalwareSmarts (talk) 21:16, 14 April 2008 (UTC)[reply]

Anesthiologist exicting?

How dynamic is the job of an anesthesiologist? After anesthetizing the patient at the beginning of an operation, the anesthesiologist's job is to continually monitor the patient and ensure that he/she is properly anesthetized. But this seems like a very passive role and during, for example, a 6 hour surgery would this be, for lack of a better word, boring? Acceptable (talk) 16:35, 13 April 2008 (UTC)[reply]

Well it wouldn't be my cup of tea, but you'd be kept pretty busy according to anesthesia#Anesthetic monitoring. Also it does get quite exciting when the patient for whom you are responsible has a cardiac arrest.--Shantavira|feed me 17:31, 13 April 2008 (UTC)[reply]
I suspect that there are a lot of highly-paid, highly-skilled jobs that are similar. The canonical description of such occupations is usually along the lines of "long periods of boredom punctuated by moments of intense panic". Airline pilots are in the same boat, as are the folks who operate nuclear reactors. Most of the time the work that they do is pretty much by rote, and often with a large degree of automation. The highly-educated, highly-trained person is very nearly redundant for much of the time, but is absolutely essential during those (hopefully infrequent) times when something goes awry.
Boredom is absolutely a concern. There are cases in the literature of both anesthesiologists and pilots falling asleep on the job (Google phrases like "anesthesiologist fell asleep" to find some malpractice lawsuits). TenOfAllTrades(talk) 20:26, 13 April 2008 (UTC)[reply]
The story is told that Michael Caine was talking to Valerie Perrine on a movie set. When making a movie it often happens that everyone else has to wait around while one specific group of people are busy doing their thing to prepare for the next shot. She asked him how it was that the waiting didn't drive him up the wall. And he said, more or less, "You don't understand: this is what they pay me for. I'd act for nothing."
The point is that most jobs are boring most of the time, and one of the reasons you get paid is so you'll keep working even though it's boring. If you can find any sort of work that you're competnant at and where even the routine stuff is kind of interesting to you, then that's the work you should be doing. But don't expect any profession to be exciting all the time. There's no such thing. --Anonymous, 22:25 UTC, April 13, 2008.
You might have the personality that likes attention to detail. As you'd be a trained medical doctor to become an anesthesiologist, I guess you'd have other options if it turns out to be too low-key for you. Julia Rossi (talk) 11:39, 14 April 2008 (UTC)[reply]
I, for one, find anesthiologists to be boring...they always put me to sleep. StuRat (talk) 16:37, 14 April 2008 (UTC)[reply]

Gold medals - Sports

Is olympic or other sport gold medals 100% gold? what is the purity and weight of these medals? What is the most valuable gold medal in the world? —Preceding unsigned comment added by HJB1 (talkcontribs) 16:46, 13 April 2008 (UTC)[reply]

In general, gold medals generally aren't gold, as far as I understand. This Google Answers result suggests that the Olympic gold medal is gilded with no less than 6g of gold. The Nobel Prize medal is, fairly famously, made of solid gold (although that's, obviously, not a sports prize). Angus Lepper(T, C, D) 17:00, 13 April 2008 (UTC)[reply]

Personal Question - please help

Hello everyone. I stumbled upon this site a few months ago only today I found this question area, so I am going to go ahead a ask a question. I have a personal problem which I need advice to, but I can't talk to my family and its not something which I want to discuss in person with anyone. I would really appreciate some feedback because I am going out of my mind with this. When I masturbate I often think of my sister. I am 17 years old and she is 25. I need to know if this is wrong or does everyone at some point fantasize about their sister? Its not like I'm obsessed with her but this has happened on more than one occasion and afterwards I feel worried that I am messing myself up mentally by doing this. Please help. Mcflyfan101 (talk) 18:01, 13 April 2008 (UTC)[reply]

First of all, let me say I really admire you for admitting to your worries and posting your question here. Secondly, this is a reference desk, not a forum for giving advice. All we can do is point you towards sources. I googled for terms I don't usually use (try "incest fantasies" or "teenage masturbation worries") and found question posted at Ask MetaFilter: "Are incest fantasies common? Are they healthy and acceptable?" The consensus seems to be that fantasies are fine in and of themselves, but that if the guilt they cause is distressing you, you may wish to seek professional advice. Certainly fantasies of one sibling for another are common. You ask if everyone has these fantasies: "everyone" doesn't do any sexual behaviour, but every sexual behaviour has been done by someone. See our articles at masturbation, sexual fantasy, and human sexual behaviour. Good luck! BrainyBabe (talk) 22:08, 13 April 2008 (UTC)[reply]
I would say there is no instant cure for this. Neal (talk) 22:10, 13 April 2008 (UTC).[reply]

And I would stay just stop masturbating.71.142.208.226 (talk) 22:52, 13 April 2008 (UTC)Cardinal Raven[reply]

You may as well say stop breathing. Really, that wasn't very sensible advice, your avian eminence. -- JackofOz (talk) 23:15, 13 April 2008 (UTC)[reply]

I'm helping him with his trauma of seeing his sister. I have no problem with someone masturbating, but that is his private business. He shouldn't be putting it on the internet because he is afraid to tell the people he knows. What are we going to do about it? We aren't going to make him feel better. So he should just stop masturbating.Btw, I'm not breathing right now.71.142.208.226 (talk) 23:29, 13 April 2008 (UTC)Cardinal Raven[reply]

I'd just say that if you have nothing worthwhile to contribute to a question, it's best to hold your tongue rather than give unhelpful answers. Stopping masturbating is avoiding the issue, not even remotely dealing with it. -- JackofOz (talk) 02:32, 14 April 2008 (UTC)[reply]
Why is the refdesk turning into a self-help site instead of a help site? and doesn't this ever so slightly tip over into the medical (psycho/logy) advice thing? Or is it feeding time at the zoo and I don't wanto work at the zoo. Julia Rossi (talk) 02:46, 14 April 2008 (UTC)[reply]
If you're 17 and the weirdest fantasy you have involves your sister, consider yourself lucky. When you start having fantasies about dead, maggot-covered racoons, then it's time to seek professional help. StuRat (talk) 16:32, 14 April 2008 (UTC)[reply]
These are emotions we all have at some stage of our lives. It doesn't always get easier as you get older either. You may occasionally still get the same feelings later in life. The way you deal with them now will set the foundations for your future. Do it wisely and think through everything you do before you do it. Remember thoughts hurt no one and no one will ever know your thoughts unless you voice them. THAT would be a silly thing to do. Good luck and I wish you well. 79.75.191.39 (talk) 17:19, 14 April 2008 (UTC)[reply]

What exactly happens when you say "Well hello, newly constructed neuropathway!"?

<moved to science desk>

Why?

Why? —Preceding unsigned comment added by VIVID (talkcontribs) 20:05, 13 April 2008 (UTC)[reply]

Identical posting at humanities desk / timestamp 20:04. --Cookatoo.ergo.ZooM (talk) 20:25, 13 April 2008 (UTC)[reply]

moved from humanities desk:<<Why? VIVID (talk) 20:04, 13 April 2008 (UTC)[reply]

Because. Daniel (‽) 20:12, 13 April 2008 (UTC)[reply]
Why not?>>
Why not? Useight (talk) 20:27, 13 April 2008 (UTC)[reply]

Why? Why?! Why? X.x71.142.208.226 (talk) 21:15, 13 April 2008 (UTC)Cardinal Raven[reply]

Because, because, be-ecause. Good night, good night. Julia Rossi (talk) 22:29, 13 April 2008 (UTC)[reply]

Because of the wonderful things he does. —Preceding unsigned comment added by 96.227.23.114 (talk) 02:06, 14 April 2008 (UTC)[reply]

? ! hotclaws 06:59, 14 April 2008 (UTC)[reply]

Researching background on William Malcolm

William Malcolm is generally credited with inventing the modern telescopic rifle scope in the late 19th century. Can you point me to a source for biographical information and info on his company and products? —Preceding unsigned comment added by 70.145.69.187 (talk) 21:14, 13 April 2008 (UTC)[reply]

Oh wow, a Google search showed results! Should I copy and paste the links, or can you go from here? Neal (talk) 22:08, 13 April 2008 (UTC).[reply]

Wow! Google actually works.71.142.208.226 (talk) 23:26, 13 April 2008 (UTC)Cardinal Raven[reply]

Condoms

So...this may sound weird but when you wear a condom...where do you cum? I just always kinda have this fear in the back of my head that if i cum in the condom while i am still in her...that maybe there will be a hole and get the girl pregnet.

what do you guys do? (this is serious question)

also...what condoms do you prefer/use? —Preceding unsigned comment added by Virgin McDumb (talkcontribs) 21:56, 13 April 2008 (UTC)[reply]

You cum right in the condom of course. If there is a hole, then you could know and verify it beforehand. I have no experience with condoms, so N/A. But since your username is Virgin McDumb, this is a contradicting fear of past experiences if you are a virgin of course. Neal (talk) 22:02, 13 April 2008 (UTC).[reply]
Condoms can indeed have holes in them, though if you have purchased them yourself in a box and cared for and used them correctly then it is unlikely. As for ejaculation, it takes place in the condom itself and goes into a little fluid reservoir at the tip of the condom. Condom packages come with instructions on them, they're worth a look over if you're planning on using them and are really uninformed on the subject. Contraception is useless if you aren't using it properly. --Captain Ref Desk (talk) 22:14, 13 April 2008 (UTC)[reply]
That user name of yours isn't going to wear well, btw. And a pharmacist will explain how a condom isn't a preg-net. They talk to dumb mcvirgins every day. Julia Rossi (talk) 22:25, 13 April 2008 (UTC)[reply]
A similar question was asked last month here. BrainyBabe (talk) 23:04, 13 April 2008 (UTC)[reply]
The bottom line is that you can never regard a condom as a fool-proof answer to pregnancy, so your fear is justified. Creating new life is no trivial matter. Vranak (talk) 01:28, 14 April 2008 (UTC)[reply]
here's a fool-proof way of avoiding unwanted pregnancy.--Lenticel (talk) 05:09, 14 April 2008 (UTC)[reply]
Sure, if you're a loser. Of course, abortion is always an option and if you're not a wimp, guilt-free, too.--71.108.64.72 (talk) 05:25, 14 April 2008 (UTC)[reply]
This method isn't as fool-proof as the above, but it's close, and you don't have to mess about with any rubber things on your cock either ^^ Kuronue | Talk 14:24, 14 April 2008 (UTC)[reply]

Need help with frend

Ok i was at YMCA swimming pool and me and this girl are good friends. So outside she was saying like we should go to family change room. I was wondering why, but i didnt question her. So when we entered i went to the change room, and she said she will stay outside at the lockers and change. Because nobody was there. And after i changed into my swimming trunks i jumped out of the changing booth and bam~ she was standing there exposing her but. Then she turned around to get something out of her swimming bag. and i saw her front body parts. It was my first time seeing a naked girl in real life, and i was shocked that her VAGINA had hair on it, and i didnt have any hair since im a late bloomer. She screamed at me and i ran out and said sorry on msn. And she wont even talk to me anymore! how do i get her to be my friend again...i dont want our friendship to be broken just cuz i saw her with fully naked??--Need help with friend (talk) 22:01, 13 April 2008 (UTC)[reply]

Sigh, who cares. Neal (talk) 22:03, 13 April 2008 (UTC).[reply]
It's a set up/accident/big nothing, get outta there, write a short story about it for your "Wow" book. Julia Rossi (talk) 22:20, 13 April 2008 (UTC)[reply]

This isn't a forum. You bound to see a naked girl some time in your life. Go bother someone who actually cares.71.142.208.226 (talk) 22:54, 13 April 2008 (UTC)Cardinal Raven[reply]

How about showing a little civility, Neal and Raven? Clarityfiend (talk) 01:19, 14 April 2008 (UTC)[reply]
So you know, hair is perfectly normal on a girl's vagina. Not ever girl wants to look like a porn star badly enough to go through the pain of waxing. Kuronue | Talk 14:26, 14 April 2008 (UTC)[reply]
No kidding! I'm no expert on such matters but why not call Dr. Laura or Joy Browne? Vranak (talk) 01:24, 14 April 2008 (UTC)[reply]
on a side note, for this bunch in here the best thing to call a vagina is "pipe dream". 79.75.191.39 (talk) 17:57, 14 April 2008 (UTC)[reply]

kiwi's name

Where did kiwi's name come from? Did it come from its call, or did it come from its reletive,the kivi?Jamiedogpark (talk) 23:02, 13 April 2008 (UTC)jamie[reply]

It's from the Maori word kiwi. According to the American Heritage, it might be of imitative origin, so maybe from its call? bibliomaniac15 Hey you! Stop lazing around and help fix this article instead! 23:10, 13 April 2008 (UTC)[reply]
*sigh* : ) Julia Rossi (talk) 23:16, 13 April 2008 (UTC)[reply]

hi, its me again!

Hi, this is jamiedogpark,and I want to know if I got any answers yet.^_^ —Preceding unsigned comment added by Jamiedogpark (talkcontribs) 23:08, 13 April 2008 (UTC)[reply]

Welcome to the refdesk Jamiedogpark. Since it's a long way from your questions, I've quoted the notice introducing the page:

*Be patient. Your question probably will not be answered right away, so come back later and check for a response. Questions are normally answered at the same page on which they were asked. A complete answer to your question may be developed over a period of up to four days". Thanks for being keen, Julia Rossi (talk) 23:13, 13 April 2008 (UTC)[reply]

Have you seen our superb Kiwi article? Julia Rossi (talk) 23:19, 13 April 2008 (UTC)[reply]
To be fair, I checked it out, and didn't see any etymology. BrainyBabe (talk) 23:46, 13 April 2008 (UTC)[reply]
Sori, so impressed by the article, missed that bit. My pop-up dictionary gives "ORIGIN mid 19th cent.: from Maori." How would I reference that in the article? Julia Rossi (talk) 00:46, 14 April 2008 (UTC)[reply]
Don't worry I've worked it out and discovered a new word that might be useful for a NZ page: Kiwikiwiki. : ) Julia Rossi (talk) 00:51, 14 April 2008 (UTC)[reply]

Stealing from robbers

Let's say someone is being mugged in a back alley in New York City. Two people are mugging the guy, stealing his stuff, etc., but the guy manages to fight back quite well, and incapacitates the crooks. Would it be illegal for the man to steal things from the crooks which did not initially belong to him? What would he be charged with? If more clarification is needed, I'll be happy to give it. Thanks. 70.105.164.43 (talk) 23:20, 13 April 2008 (UTC)[reply]

  • He could be charged with theft, of course; he took stuff that didn't belong to him. In actual fact, he probably wouldn't get charged, but only because the muggers probably wouldn't want to involve the police. Matt Deres (talk) 23:36, 13 April 2008 (UTC)[reply]
In which world would that be right morally? There is more to life than whether it is illegal or not.hotclaws 07:02, 14 April 2008 (UTC)[reply]
I believe this was a major part of the theme of the movie What's the Worst That Could Happen? starring Danny DeVito and Martin Lawrence, although there the valuable (a ring) was obtained by merely lying to the police that apprehended the thief and telling them that it was stolen. StuRat (talk) 15:43, 14 April 2008 (UTC)[reply]

Fails in Comparison

Me and my friend have a review site in which we review video games. So I decided review Need For Speed: Pro Street for PS3, PS2, 360, and PSP (which everyone had been anticipating the PSP port). So when I bought it one week ago, I popped it into my PSP and saw this low res. car greeting me. Thinking I was seeing things I popped in my PS3 version and blamo! it was beautiful. Then I went back to my PSP and how disappointed I was. The level design was anything BUT spectacular, the controls were clunky, and worst of all there was NO story, just this lame career mode with events you do to unlock stuff. Also there was no body kits, no autosculpt, and NO vynls. After beating the whole game in about an hour I thought maybe I should check online, only three people were online and not ONE of them wanted to play. The PSP port was like EA just grabbed the name of the title and crapped in my UMD disc and sold it for 40$. This version didn't bring anything good to the NFS franchise but shame and humiliation, and I suggest anyone that has this game hurry and trade it in for the PS2 or PS3 version immediately!I eventually gave this crap game a 1 out of 5 for being the worst game on the PSP. What the hell is with this half ass porting? I've seen full auto games that deliver more than this bland piece of crap. --SlaveofBetrayal (talk) 23:47, 13 April 2008 (UTC)[reply]

Did you have an actual question? I'm afraid I can't see what it is you would like to know. 79.66.105.94 (talk) 00:03, 14 April 2008 (UTC)[reply]


I think its mostly to do with money. The companies want more money. They know someone is willing to pay 40 bucks for a game that they don't know is half done. Its a win situation the customer thinks its NFS, but it isn't. The same problem with Katamari Damashi for the PSP. Its very hard to work with. I hate it I like the dual analog sticks not press the up button and the x-triangle buttons. The PSP is a wonderful machine some games are okay, but others are just plain and boring.

Always

Cardinal RavenCardinal Raven (talk) 00:05, 14 April 2008 (UTC)Cardinal Raven[reply]

Whoops. My question is: Why can't developers make the same quality game for all version and ports?--SlaveofBetrayal (talk) 00:06, 14 April 2008 (UTC)[reply]

They do make the same game for all the versions ports. They just do something silly with the ports. Either they add in unnecessary fluff or to little fluff. Also because that is what it is. Its just a port. Thats all it is.71.142.208.226 (talk) 00:54, 14 April 2008 (UTC)Cardinal Raven[reply]


Um can I clarify something? NFS Carbon is waaaaay different from NFS Carbon on the PSP. Same with other games. The God of War game (famous with Play Station owners) for the PSP fails in comparrison to its predecessors God of War and God of War 2. I'll tell you what's wrong with the PSP version... ready? Clunky controls (not as smooth as its console counterpart). Also as you stated before the Katamari game on the PS2 is way better in comparison with the PSP version AND 360 version. Also another detail that game companies either add too little or too much. But in most cases for handhelds they seem to make the game terrible. Well FF Seven Crisis Core wasn't a bad game it did have its repetitive game play and random encounters.--SlaveofBetrayal (talk) 01:04, 14 April 2008 (UTC)[reply]

Of course its clunky controls. They don't have the analog anymore. I loved the dual analog. They port these games and keep the same system from the PS2 to the PSP.71.142.208.226 (talk) 01:08, 14 April 2008 (UTC)Cardinal Raven[reply]

Why hasn't anyone said the true answer? The PSP has no where near the processing and rendering power of the PS3, and the UMD has no where near the storage space of a Blu-Ray disc. It is impossible to have the same game on both platforms. The PSP one will obviously be forced to be smaller and simplier. 206.252.74.48 (talk) 13:13, 14 April 2008 (UTC)[reply]

Strength of the American Black Bear compared to humans

After reading the article on fatal bear attacks in the U.S., i got kind of curious about how an above-average human would do in a fight against an American Black Bear. I'm talking someone really massive, such as Shaq or The Great Khali. Is it physically possible for a human, even a 7-foot, 300 or 400 pound human, to fight off a bear in the wild? And when I ask this question, I mean a kind of desparate, hungry bear who isnt going to run off from one solid punch to the head (unless of course, that is their nature no matter how hungry they are). I realize this is a weird question, and the answers will be little more than speculation, but who knows who on here knows what they're talking about. Thanks. 70.105.164.43 (talk) 23:52, 13 April 2008 (UTC)[reply]

Have you ever been threatened by a dog? If you have, you'll know that punching it in the head just doesn't work -- they move and react too fast. Probably the same with a bear. Wild things are just more comfortable in their skins than us humans. And punching a bear in the head would probably hurt you more than it -- their skulls are very solid in order to give structural support to their strong jaws. And provoking a bear by punching it is probably not wise. Better to keep a respectful distance in the first place, be submissive and move away if they grow perturbed, and play dead if they attack. If a bear does attack, he's already sized you up and decided he can beat you. Vranak (talk) 01:21, 14 April 2008 (UTC)[reply]
Around feral dog packs, I was warned to be submissive. Luckily didn't have to test it. Aggression invites a no-contest as per Vranak's "in their own skins". I'd guess even famous wrestling bears (Rocky?) are trained to hold back somewhat. Julia Rossi (talk) 01:56, 14 April 2008 (UTC)[reply]
I bet Goliath could've taken out a bear. Or Samson. Or Andre the Giant. Useight (talk) 05:12, 14 April 2008 (UTC)[reply]
Nature gave us brains rather than defensive or offensive body structures. Too bad we can't bludgeon them with our brains. In a bare-handed contest, we'll lose. Human skin isn't that hard to pierce and human muscle is not as strong as those of the creatures of the wild. However with the right tools and strategy, even a small human can defeat bears or lions.--Lenticel (talk) 05:43, 14 April 2008 (UTC)[reply]

I watched a documentary about tigers. The female tiger chased down a bear. The bear ran scared for its life. So a Tiger could take down a bear. The documentary said that bears on the tiger's lunch menu.71.142.208.226 (talk) 05:41, 14 April 2008 (UTC)Cardinal Raven[reply]

It seems that bears and tigers have an interesting relationship.--Lenticel (talk) 05:46, 14 April 2008 (UTC)[reply]
Bears? Humans don't even stand a chance against chimpanzees! The "average chimpanzee has over 5 times the upper-body strength of a human male". Dunno about Oprah though. Clarityfiend (talk) 08:49, 14 April 2008 (UTC)[reply]
I don't know about Goliath and co, but Chuck Norris beat a bear in a fight quite handily, I believe. And I do know that Timothy Treadwell couldn't handle a bear, although he was probably outnumbered and a little bit nuts. Michael Clarke, Esq. (talk) 13:52, 14 April 2008 (UTC)[reply]
My two cents? A black bear could walk right up to and simply start eating the best human fighter ever known without even dealing with his futile attempts to defend himself. Look at the best human-on-human fighters, say the Brazilian jiu-jitsu guys or the sambo guys. They often take several minutes to hurt one another much, and the fight usually goes to the ground right away. Bears go on all fours and can't be knocked down, whereas we don't and certainly would be, so we're really talking about human ground game vs. ursine ground game, a dismal prospect for the human. Too, it's important to bear (pun) in mind that a human has no way of crushing bone or ripping off massive chunks of tissue and swallowing them before the horrified eyes of his opponent. The sort of damage a bear can do takes the starch out of a human pronto; once the man started to lose, and start to lose he would in short order, death would ensue in seconds. I'm going with flight or bluff if I'm ever at odds with a bear and unarmed, but I don't expect ever to be. I'm with Stephen Colbert and Ted Nugent. Bear-o-phobia makes strange bedfellows. (By the way, did you know that Winnie the Pooh is a black bear? Makes you think.) --Milkbreath (talk) 15:03, 14 April 2008 (UTC)[reply]
One general comment on predators: Any injury can make them unable to hunt and result in their death from starvation. As an adaptation to this threat, they're often far more timid than their capabilities would seem to indicate. When confronted with even a modest threat, they often back down, rather than risk injury. Even if confronted with the unfamiliar, they may think "what the heck is that ? I don't know, so I'd better play it safe and attack something I'm sure won't injure me". Tigers, for example, can be kept from attacking people merely by having the people wear a mask on the back of their heads. They always attack from the rear, since it's safer, and won't normally risk a frontal attack, even against a puny human. StuRat (talk) 15:27, 14 April 2008 (UTC)[reply]
When I saw that that link was to "hunting Ted Nugent", I really got my hopes up... Michael Clarke, Esq. (talk) 18:03, 14 April 2008 (UTC)[reply]
Do you, by any chance, support the "right to arm bears" ? StuRat (talk) 20:32, 14 April 2008 (UTC)[reply]
I must confess I've never read them so I can't claim to be an avid supporter, but the concept has indeed got me interested. Michael Clarke, Esq. (talk) 20:59, 14 April 2008 (UTC)[reply]

April 14

Semen

Okay, my sex life isn't exactly suffering but it COULD be improved... my boyfriend's semen is actually tasty, apparently because he eats a lot of fruit. Mine is salty and disgusting... what makes semen taste better, in terms of food? I'm not a huge fan of fruit so is there anything that I can eat to improve the epicurean appeal of my semen?

-apologies if this sounds weird, i'm trying to make this question sound as unoffensive as possible —Preceding unsigned comment added by 66.220.246.235 (talk) 01:35, 14 April 2008 (UTC)[reply]

There are online talk fests that relish this stuff? Wikipedia is not one of them (no disrespek). Julia Rossi (talk) 01:50, 14 April 2008 (UTC)[reply]
Unless a Reference Desk editor happens to be an expert on this topic, we have to rely on published sources. I haven't been able to find any reliable research on this question. However, you can find sources on semen flavor if you search Google. I feel obliged to point out that semen in your partner's mouth (or any other body orifice) can transmit sexually transmitted diseases. Marco polo (talk) 02:10, 14 April 2008 (UTC)[reply]

Oh, we're both clean, by the way. —Preceding unsigned comment added by 66.220.246.235 (talk) 02:26, 14 April 2008 (UTC)[reply]

There is a common urban myth that fruit juice (particularly pineapple, for some reason) is good for taste, but I haven't had any evidence either way (never tried!). I'd stick with it being an urban myth, unless someone else is willing to google (not me) about it. Steewi (talk) 03:27, 14 April 2008 (UTC)[reply]
Playboy gets this question every so often in their "Advisor" column. You might try your local library for back issues. Dismas|(talk) 04:19, 14 April 2008 (UTC)[reply]

Is Yahoo Answers off line? Looking at the latest spate of questions, I'm begining to think so.hotclaws 07:03, 14 April 2008 (UTC)[reply]

By a strange coincidence yes it was down this morning for a couple of hours for maitainance, clearly this was an urgent question for the OP. —Preceding unsigned comment added by 86.4.187.55 (talk) 13:06, 14 April 2008 (UTC)[reply]

I don't know if this is true or not, but in the 4th season finale of Curb Your Enthusiasm, an alleged 'blowjob expert' reports that spicy food makes semen taste good. Keep in mind this is a fictional sitcom. The Guide To Getting it On by Paul Joannides suggests popping a mint or a dab of touthpaste in your mouth prior to doing the deed to mask flavor. -Shaggorama (talk) 09:05, 14 April 2008 (UTC)[reply]

FOLLOWUP: it took me a little browsing to find a more pertinent entry in 'the guide,' but as i suspected one exists. the guide (as cited above) suggests the following substances may effect semen flavor, although this is based entirely on 'what they've heard,' so don't take any of this as fact.
  • stuff that may make semen taste bad: red meat, coffee, cigarettes, stress
  • stuff that may make semen taste good: celery, fruits
hope this helped--Shaggorama (talk) 12:21, 14 April 2008 (UTC)[reply]

Ear canal as a body orifice

Sorry, but I just have to ask; in regards to Marco polo’s warning in the above question that semen in body orifices can transmit STDs, can semen in the ear canal do the same? (Yes, this is a serious question, and no, let’s not speculate on how it could happen.) --S.dedalus (talk) 03:12, 14 April 2008 (UTC)[reply]

Serious as it might be, the speculation would be far more interesting.--Artjo (talk) 09:21, 14 April 2008 (UTC)[reply]
Two Three things, one, how could something reproductive be small enough to enter an ear canal let alone transmit STDs; and two, the next time someone licks you in the ear to unnerve you at a party, best ask where that tongue has been. Julia Rossi (talk) 10:05, 14 April 2008 (UTC)[reply]
And three, what about a nostril? Julia Rossi (talk) 10:07, 14 April 2008 (UTC)[reply]

What sort of parties do you go to? Lemon martini (talk) 11:31, 14 April 2008 (UTC)[reply]

A martini party?  ; ) * Julia Rossi (talk) 12:05, 14 April 2008 (UTC)[reply]
In response to Julia's first response, i'm positive ejaculate could reach as far as the tympanic membrane without the penis actually being inserted into the ear canal. SPECULATING, most STDs probably cannot be transmitted by this pathway, as the eardrum does not constitute a mucuous membrane. The nostril does, although I dont want to imagine how semen would end up in there.. Furthermore, there are some STDs that still might possibliy be passed through/to an ear, such as HPV (warts). WHAT A STRANGE QUESTION. Shaggorama (talk) 12:15, 14 April 2008 (UTC)[reply]
"Ok, but only the left ear...I'm saving the other for Mr. Right." StuRat (talk) 15:17, 14 April 2008 (UTC)[reply]
On examining the above post by Shaggorama, after carefully working my way through the haze of spelling errors, abuse of capitalization, and general crimes against grammar and syntax, I have demonstrated that, beneath it all, the work betrays the author's staggering ignorance of the history and the workings of sexually transmitted diseases. While the author's wildly swerving train of thought did at one point flirt with coherence, this brief encounter was more likely a chance event than a result of rudimentary lucidity. 79.75.191.39 (talk) 17:55, 14 April 2008 (UTC)[reply]

When "you" think, what thinks?

Science has shown that thoughts appear in your mind before you acknowledge them (and take credit for it). When you think, "Hmm...I feel the need to drive across town and touch my dad's mailbox. Tonight at 2am? By golly, I'll do it!" The thought "appeared", including the thought to respond to the original thought.

Right now your mind is thinking. You have little or no control over it. You "think" you do, but again, those are just thoughts "appearing". What is it that is programming our minds? What is the phantomlike presence(s) that create conscious and unconscious thoughts, alpha states, dreams, and bursts of inspiration?--Sam Science (talk) 03:11, 14 April 2008 (UTC)[reply]

Subconscious...maybe.71.142.208.226 (talk) 03:42, 14 April 2008 (UTC)Cardinal Raven[reply]

I think the research you're referencing is that of Benjamin Libet. I think the standard interpretation is that his research showed that our thoughts must be the result of material processes, with the only room for free will being "veto power" over whether or not to respond to the original impulse. Personally, I think this usual interpretation conflates phenomenal consciousness with access consciousness. I think Libet's experiments only show that we are not conscious of our thoughts immediately; that is, we do not immediately have the secondary thoughts necessary to identify and report on the primary thought. But I don't see how that shows that the primary thought wasn't a conscious one, in the sense of phenomenal consciousness. So, while I think your question is a good one, I don't think Libet's research, on its own, forces us to look outside free will for the cause of our thoughts. --Allen (talk) 04:57, 14 April 2008 (UTC)[reply]
What is it that is programming our minds? -- Lots of things. Sensation and perception for one. You are, after all, alive and experiencing the world around you. Stuff comes in, stuff comes out. There's a funny metaphor about the brain being a gland that secretes thought. Body and mind respond to stimulation. What else? Emotions, habit, conditioning of one kind or another, the desire to avoid pain, the desire to feel better, and so on. The general topic is addressed a bit at Decision making ("who" decides?) and Executive functions. Finally, you may have more control than you think -- but I'm not sure how to address that topic offhand, without tumbling into either cliches or mysticism, so I won't try. I'll just suggest that while this kind of reasoning might lead to the abandonment of responsibility ("you" didn't do it, it just happened!), that would be immoral. Pfly (talk) 06:55, 14 April 2008 (UTC)[reply]

Designing weapons

How do countries like Russia/China develop advanced weapons like fighters and tanks comparable to the US with only a fraction of the research budget? I know wages are lower in these countries but those savings can't account for the massive budget gap. So what gives?

Lotsofissues 04:13, 14 April 2008 (UTC)

Maybe they reverse engineer existing US weaponry and improve them rather than start from scratch.--Lenticel (talk) 04:45, 14 April 2008 (UTC)[reply]

Or perhaps, hard as it might be to believe, they use their own ingenuity. You may be surprised to find that not everything has been invented or perfected by Americans, and indeed a reduced budget might well focus one on the task rather than gold plated toilet seats and thousand dollar spanners 125.237.93.6 (talk) 07:12, 14 April 2008 (UTC)[reply]
We don't spend our money on $1,000 spanners; we spend it on $1,000 wrenches. (Or hammers, I forget which it was this week.)
Atlant (talk) 12:14, 14 April 2008 (UTC)[reply]
An anecdotal example of some dubious veracity:
I read many years ago that the USA / NASA invested a significant budget in the development of a writing tool which could be used in zero gravity by astronauts.
The Russian equivalent of the Space Agency decided, that the good old-fashioned lead pencil would do the trick quite nicely for their own cosmonauts. --Cookatoo.ergo.ZooM (talk) 08:21, 14 April 2008 (UTC)[reply]
Ahem, it's a little on the urban L side as you rightly sussed Cookatoo. Take a peek at this[33] in the article Space pen. Julia Rossi (talk) 08:41, 14 April 2008 (UTC)[reply]
Sometimes they just do it, as the Night Witches who flew almost 20-year old wood and canvas biplanes on night bombing missions, carrying only two bombs at a time. Roughing it and improvising was part of the life on Mir, the Soviet space station. Julia Rossi (talk) 09:11, 14 April 2008 (UTC)[reply]
Well, there have been a number of famous examples. --Sean 12:43, 14 April 2008 (UTC)[reply]
I would need some evidence that China has produced a weapon comparable to those in the West; I don't think they have. I'm not sure "Russia" has, either. The only test of the relative quality of two weapons is to pit them against each other. This has not happened with aircraft and ships, only with tanks, and the Russian ones got creamed like they were apple-crate go-carts. Sure, some F-15s have shot down some Mig-29s, and some F-14s have wiped some Mig-23s, but they were unfair contests. We haven't seen Eagle vs. Flanker, which would decide the title. As for ships and subs, a look at the technical abilities of a U.S. carrier battle group or a Seawolf class submarine suggest that their worthiest opponents wouldn't last five minutes in a real war. --Milkbreath (talk) 10:55, 14 April 2008 (UTC)[reply]
George W. Bush seems to be trying to arrange a full-scale test of this stuff, but I'm not as sanguine as you are about the survival of our carrier battle groups. It seems to me that the U.S.S. Cole, combined with the Turtle proves something about what can be done by dedicated combatants. A truly massive simultaneous attack of low-tech, cheap missiles might also prove something, Phalanx guns notwithstanding. But GWB doesn't learn from anyone's mistakes, his or anyone elses...
Atlant (talk) 12:14, 14 April 2008 (UTC)[reply]
Wikipedia is not a soapbox. The foreign policy of the current administration of the US has no bearing on the question, nor does the potential efficacy of non-traditional warfare tactics. This is not the place to bash Bush or congratulate the Yemeni terrorists, both of which cants betray extreme POV and invite off-topic rebuttal. Please confine your remarks to well-referenced direct answers to questions. --Milkbreath (talk) 13:44, 14 April 2008 (UTC)[reply]
And note that the Cole was only vulnerable to that type of attack because they were not prepared for it. Such an attack typically only works once. US ships now shoot anyone who approaches in a boat and refuses to identify themselves. StuRat (talk) 15:03, 14 April 2008 (UTC)[reply]
Perhaps an important part of the price difference is that the US, which has a tragically politically powerful military industrial complex to support, is happy to spend well past the point of diminishing returns. --Sean 12:35, 14 April 2008 (UTC)[reply]
Perhaps the smaller budgets lead to an incentivisation of creativity and novel solutions, in order to circumvent budgetary requirements. Agreed with Sean about the military industry in the US, they have powerful connections and there are only about 3 companies that can realistically compete for military contracts, so prices will remain sky-high, especially because of America's willingness to spend. Countries like Russia hardly have small weapons budgets, though. Michael Clarke, Esq. (talk) 12:59, 14 April 2008 (UTC)[reply]

It's actually an incredibly interesting question. They have very different funding models for the military and very different priorities in terms of weapons. In the US, for example, there has long been a willingness to reinvent the wheel each time and and a belief that being along the technological cutting edge is of primary importance; in Russia, by contrast, there is a long tradition of using preexisting designs and preexisting standards, on interchangeability of parts, of them being easy to repair. The old Soviet idea was to make up the potential deficits in quality with an increased quantity, and a recognition that cutting edge has its drawbacks as well from a production and maintenance standpoint. Interesting article on the subject: Leon Trilling, "Styles of Military Technical Development: Soviet and U.S. Jet Fighters - 1945-1960," in E. Mendelsohn, et al, eds., Science, Technology and the MIlitary (Springer, 1989). As for the old reverse-engineering saw, it's important to remember that it isn't like the Soviets would ever just take something and copy it exactly without re-testing every part on it. Their atomic bomb project is supposed to be the canonical example of their copying, but of course they didn't just say "Oh hey, this is great, let's just duplicate every part on it exactly! We totally trust our intelligence sources!" In reality the administrators kept the intelligence data as a "check" on their own scientists and used it as a way to direct research to areas they knew had already worked for the Americans. Most of the scientists working on the Soviet bomb had no idea that the final product was a copy and felt they had developed it all independently; in a way, they did, in a way, they didn't. It still required a lot of hard work and ingenuity, and they ended up doing a great deal many things that wasn't in the intelligence in any case (the primary Soviet achievement, one that the US did not see coming, was in its use of low-grade uranium ores; this is what made their project get finished quickly, not the intelligence). --Captain Ref Desk (talk) 13:14, 14 April 2008 (UTC)[reply]

By the List of countries by military expenditures and List of countries by population USA has only a military money / population -ratio of 1/1 = 1, but my mighty home country Finland gets a massive 111 / 41 ~ 2.7 score. Next Greater Finland will probably cover the whole Europe. --212.149.216.233 (talk) 14:32, 14 April 2008 (UTC)[reply]
There are examples of direct copying, however. In the civilian aircraft market the Soviets directly copied the British/French Concorde to produce the Tupolev Tu-144. (They did make some changes because they couldn't duplicate all the features of the Concorde, like it's engines and wing curvature.) StuRat (talk) 14:59, 14 April 2008 (UTC)[reply]

Transportation---- truck/size /weight regulations--Michigan

I am trying to establish the heavy truck size & weight limits for Michigan(USA) from 1956 to 1974 for heavy Michigan multi-axle sand/gravel dump truck/trailer/combinations. I have used several search engines now without success in my research. Have any members out there access to or know where to obtain this information please, which seems to be very elusive. —Preceding unsigned comment added by Coljenks (talkcontribs) 06:56, 14 April 2008 (UTC)[reply]

have you tried contacting the michigan DOT/MVA? --Shaggorama (talk) 09:18, 14 April 2008 (UTC)[reply]

trivia

trivia mostly preferred by the Americans and Asians. What is it? —Preceding unsigned comment added by 193.219.249.15 (talk) 10:17, 14 April 2008 (UTC)[reply]

Preceding question. Topic is? Kuronue | Talk 14:33, 14 April 2008 (UTC)[reply]
Well, both America and Asia have a lot of nerds and nerds love trivia. Recury (talk) 14:57, 14 April 2008 (UTC)[reply]
Damn straight we do. Did you know that 3 out of 4 people make up 75% of the world's population? 206.252.74.48 (talk) 15:23, 14 April 2008 (UTC)[reply]

is it true?

That in Tokyo, a bicycle is faster than a car for most trips of less than 50 minutes? thanks everyones —Preceding unsigned comment added by Its hard to believe (talkcontribs) 11:05, 14 April 2008 (UTC)[reply]

With the traffic, I wouldn't be surprised. Probably the same in New York City. Useight (talk) 15:34, 14 April 2008 (UTC)[reply]
This is certainly true in central Boston for most trips of less than 20 minutes (based on OR), and even central Boston is less dense than most of Tokyo, so I would expect that this would be true for trips under 50 minutes in Tokyo. (Other than at rush hour, however, I'm not sure that it would be true for trips up to 50 minutes in Manhattan, because it takes less than 50 minutes to cross the island, and because the broad avenues, the West Side Highway, and the FDR Drive allow cars to move uptown and downtown fairly rapidly.) Marco polo (talk) 19:58, 14 April 2008 (UTC)[reply]
Well, just a few asides, but Top Gear did a few tests like this. They raced a car, a bike, a boat and public transport across London, and the car came last by quite some way, with the bike first. They also raced a marathon runner against a car along the route of the London Marathon and the runner won. -mattbuck (Talk) 20:33, 14 April 2008 (UTC)[reply]

definition to describe this

dear reference desk, i have an unusual habit and was wondering if anyones else has heard of this before the fascination with hearing others passing wind is intoxicating to me. is there and definition to describe this? —Preceding unsigned comment added by Only in it for money (talkcontribs) 11:10, 14 April 2008 (UTC)[reply]

Insanity?--Artjo (talk) 11:37, 14 April 2008 (UTC)[reply]

WHAT!? You're right insanity.71.142.208.226 (talk) 13:21, 14 April 2008 (UTC)Cardinal Raven[reply]

Actually the correct term is eproctophilia, although it doesn't have a wikipedia page, or even a mention in any part of wikipedia. This was the site where I learnt this new titbit, though, and it seems to be entirely devoted to strange philias. Never call something insanity when in all likelihood it's much, much more common than you'd think. Michael Clarke, Esq. (talk) 13:31, 14 April 2008 (UTC)[reply]
Condemnant quod non intellegunt. 206.252.74.48 (talk) 15:22, 14 April 2008 (UTC)[reply]

I can call it insanity. When I hear someone cuting the wind I break out in a sprint. I don't go for their butts just to hear the noise. That is at least what I do.Cardinal Raven (talk) 16:26, 14 April 2008 (UTC)Cardinal Raven[reply]

But the OP mentions his habit being "intoxicating". Does that mean he inhales? —Preceding unsigned comment added by 92.9.254.133 (talk) 17:36, 14 April 2008 (UTC)[reply]

Youngest autobiography writer

I've tried Autobiography with not much luck-who is the youngest person to have written an autobiography?Charlotte Church was 14 when she wrote hers and I'm sure Drew Barrymore was only 13 or 14-but there must be a younger one somewhere surely? Lemon martini (talk) 11:36, 14 April 2008 (UTC)[reply]

I don't know how reliable this is, but a Drew Barrymore fansite that wikipedia won't let me link to says that Guinness World Records acknowledges her as the youngest author of an autobiography. I searched this string, and all the results are about her, so I'm guessing it is indeed Miss Barrymore. Can't find anything official from Guinness about it, though. Michael Clarke, Esq. (talk) 12:51, 14 April 2008 (UTC)[reply]
Yeah, it's not on the Guinness website for youngest people to do things, but there seem to be very few records viewable on the site. Probably saving as much stuff as possible for the book. Michael Clarke, Esq. (talk) 12:55, 14 April 2008 (UTC)[reply]


Article for credit card vs hard cash?

Do we have an article that describes the habits of different countries' population, in regards to stuff bought with credit cards as opposed to in coins or bills? I am not sure, but I think I've noticed that Americans use more bills than Norwegians do. But I'm not sure. 81.93.102.185 (talk) 15:00, 14 April 2008 (UTC)[reply]

Another breeze

Prompted by the above question about flatulence, I also have what seems like a strange compulsion. Like most people, I find the smell of farts repulsive. However, my own farts I don't find repulsive. In fact, I find them strangely compelling and sometimes even enjoy them. This is only my own farts I sniff & enjoy, I hate other peoples farts as much as the next guy. Am I a freak or is this normal? 79.75.191.39 (talk) 17:17, 14 April 2008 (UTC)[reply]

Normal.--droptone (talk) 17:31, 14 April 2008 (UTC)[reply]
See Smug Alert!. But yes, strangely enough it is very normal. Anyone know why? Michael Clarke, Esq. (talk) 18:24, 14 April 2008 (UTC)[reply]
If you upload a few samples to WP:Commons we could all have a quick sniff. --Cookatoo.ergo.ZooM (talk) 19:45, 14 April 2008 (UTC)[reply]

Tagging

One thing that I've never understood is people writing tags on walls. Apparently, a person chooses an identity by assigning a group of letters to himself, and then he tags this moniker on every wall he can think of. What does he get out of it? Who is going to know he did it? It's obvious the public won't know it, but how will even other taggers know he did it? Of course this kind of anonymous public writing can be a goal all to itself, but then there should be some kind of message, such as "Free Tibet now!" not just "some guy who you'll never meet was there". JIP | Talk 18:47, 14 April 2008 (UTC)[reply]

Not at all a complete answer : Being 'known' by a pseudonym is not that same as not being known. If I tagged up all the walls in WikiCity with some impressive work of art featuring my initials "APL" all the while never being caught by the WikiCity Police Department, then all the people in town would know of me and of my great works. The fact that no one knew who APL is and no one could pick me out of a lineup is a different matter altogether. APL (talk) 19:03, 14 April 2008 (UTC)[reply]
But would that merely mean that "APL" is famous, or that you are famous? How would you prove being "APL"? And who would you admit, and prove, it to? JIP | Talk 19:23, 14 April 2008 (UTC)[reply]
A lot of the time, tagging is reputation-based. So obviously his mates will know that he's that tag, and it's not uncommon for people to use their tag as a moniker, so perhaps their rivals will know that x-tag is y-person. It's a lot like rapping names, and often the two are one and the same, as tagging and graffiti in general often go hand-in-hand with rap music. In London, where it's mostly high-schoolers who put up tags, if you are well known then people will likely know your tag. But it works the other way, too: if you put your tag in a dangerous or noticeable place, people will ask, "who is x-tag?" and their mates will reply "oh, he's y-person". it's basically all about street-cred. I hesitate to link it with gangs, because of the "organised" perception of gangs, but their are parallels, and all of the wannabe gangsters I knew had a tag, even if they didn't go about putting it on every surface they could. So even though there isn't some sort of international tagger newsletter that keeps them up-to-date on who's who in the world of spray-painted psuedonyms, word spreads because people talk. There's also the satisfaction of walking past it and thinking "I put that there". Michael Clarke, Esq. (talk) 20:12, 14 April 2008 (UTC)[reply]

Internet rebellion

Basically, I am involved in a rebellion on an internet forum. my job is to persuade people to join, so I thought a good way to do this would be to have an interesting logo of some sort. However I have searched around the internet a lot and still can't find one. I was wondering, since so many people come here, whether someone might know somewhere where I might find one I could use. Especially useful would be one that suggests that we're together and we will win and stuff without making it look like we're evil and violent, since that's what the people we're rebelling against are like. Also I would like it if anyone here can give me any advice on how to do this, since I've never been in an organised uprising like this before. 172.202.134.122 (talk) 18:50, 14 April 2008 (UTC)[reply]

I personally only have experience with MSPaint, but I know a lot of people make fancy logos with PhotoShop. Neal (talk) 19:04, 14 April 2008 (UTC).[reply]

Finding Life Goal

I feel like I am at a dead end job in a corporation. I am bored with life. Does anyone know where online is the best career search meaning a quiz-like site to help determine what your passions are and which directions you can go to start a brand new career? --209.0.0.29 (talk) 19:04, 14 April 2008 (UTC)[reply]

No, but a small note: perhaps the 'best job' for you is no job. Seems to work alright for the every species on earth aside from one. Vranak (talk) 19:24, 14 April 2008 (UTC)[reply]
Hunting for food can certainly be a full time job. The other species just have less variation. APL (talk) 19:52, 14 April 2008 (UTC)[reply]
Your topic is about life goal whereas your paragraph is about job goal. Well, for the title, of course you have a purpose in life - to find a purpose. As per your question, I'd say find a job that makes you happy. But seriously, though, find a job where you are more qualified in. Probably ones matching what college degrees you have. Neal (talk) 19:34, 14 April 2008 (UTC).[reply]
Maybe try Monster.com? · AndonicO Engage. 19:55, 14 April 2008 (UTC)[reply]
No matter what you do, you will most likely get bored of it quite quick. I've never seen anyone who says they get up in the morning, leap right out of bed, and go to work whistling and bursting with energy and happiness. Don't try to mix your passions and your job. If you love to do something, then being forced to have to do it every day will make you hate it pretty quick. Rather, do to your workplace, do what you have to do, and then go home and do what you want to do. It works perfectly for me. 206.252.74.48 (talk) 20:04, 14 April 2008 (UTC)[reply]

historic enterprise value

Hi, I want to know whether the historic Enterprise value is taken to calculate the EV/SALES and EV/EBITDA multiples?

Say, for Ex: if I need to calculate EV/EBITDA for 2005, 2006 and 2007. I have historic EBITDA for all the three years and EV also. But, Do we actually take historic EV or latest EV for the historic multiples. I saw a Research Report having latest EV for historic multiples. I wanted to know the logic behind it.

Thanks

Santosh —Preceding unsigned comment added by Santoshhh (talkcontribs) 19:07, 14 April 2008 (UTC)[reply]

Stalemate in chess

What is the largest number of pieces it is possible to achieve stalemate with? And the fewest moves to stalemate? Do those questions make sense? I had some trouble trying to word them without taking up a few lines. Thanks. Michael Clarke, Esq. (talk) 20:17, 14 April 2008 (UTC)[reply]

The questions make sense to me, and the article on stalemate seems to answer them both. (See Stalemate#Stalemate_in_problems: It's possible to achieve stalemate with all pieces on board. 10 moves is the fastest possible way to stalemate) ---Sluzzelin talk 21:02, 14 April 2008 (UTC)[reply]
Wow can't believe I didn't think to look there. I guess I just thought that it wouldn't be there or something. Thanks a lot, this has been bugging me and some friends for a while now. Michael Clarke, Esq. (talk) 21:07, 14 April 2008 (UTC)[reply]